You are on page 1of 128

CHEMICAL REACTION

ENGINEERING

For
CHEMICAL ENGINEERING
CHEMICAL REACTION
ENGINEERING
SYLLABUS
Theories of reaction rates; kinetics of homogeneous reactions, interpretation of kinetic
data, single and multiple reactions in ideal reactors, non-ideal reactors; residence time
distribution, single parameter model; non-isothermal reactors; kinetics of
heterogeneous catalytic reactions; diffusion effects in catalysis

ANALYSIS OF GATE PAPERS


Exam Year Total
2001 24
2002 24
2003 15
2004 21
2005 17
2006 11
2007 20
2008 19
2009 11
2010 09
2011 10
2012 10
2013 11
2014 14
2015 11
2016 11
2017 13
2018 13

© Copyright Reserved by Gateflix.in No part of this material should be copied or reproduced without permission
CONTENTS
Topics Page No
1. CHEMICAL RECTION KINETICS

1.1 Introduction 01
1.2 How a Reaction Proceeds 01
1.3 Rates of a Reaction 03
1.4 The Rate Law 03
1.5 Types of Reaction System 04
1.6 Relations for CVRS & VVRS 06
1.7 Kinetic Equations for Constant Volume Reaction System 06
1.8 Kinetic Equation for Variable Density Systems 11
1.9 Half-Life of a Reactions 11
1.10 Temperature Dependency of Rate 11
Gate Questions 13
2. DESIGN OF ISOTHERMAL IDEAL SINGLE REACTORS

2.1 Introduction 24
2.2 Ideal Batch Reactors 24
2.3 Continuous Reactors 25
Gate Questions 32

3. DESIGN OF MULTIPLE REACTOR SYSTEM

3.1 MFRs In Series 45


3.2 Reactors In Parallel 47
3.3 Best Reactor Arrangement 48
Gate Questions 49

4. MULTIPLE REACTIONS

4.1 Selectivity 58
4.2 Yield of a Parallel Reaction 60
4.3 Optimum τ for a Series Reaction in a MFR 62
Gate Questions 63

5. NON-ISOTHERMAL REACTORS

5.1 The Energy Balance Equation 71


5.2 Adiabatic Reactors 72
5.3 Equilibrium and Actual Conversion of a Reaction 73
5.4 Reversible Reactions in Adiabatic Reactors 74
5.5 Optimum Temperature Progression 75
Gate Questions 76

© Copyright Reserved by Gateflix.in No part of this material should be copied or reproduced without permission
6. NON-IDEAL REACTORS
6.1 Residence Time Distribution 85
6.2 RTD Experiment 86
6.3 Pulse RTD Experiment 86
6.4 Step Tracer Experiment 89
6.5 RTD of Reactors in Series 90
6.6 RTD of Reactors Connected in Parallel 90
6.7 Non-Ideal Reactor Models 91
Gate Questions 92

7. HETEROGENEOUS SYSTEM

7.1 Non-Catalytic Reaction System 104


7.2 Catalytic Reaction System 108
7.3 Design Equation for Reactors Containing Porous Catalyst 112
7.4 Activity of Catalyst (a) 112
7.5 Deactivation of Catalyst 112
7.6 Observed rate of Reaction 113
77 Shrinking Care Model 113
7.8 Knudsen Diffusion and Knudsen transport 115
Gate Questions 116

© Copyright Reserved by Gateflix.in No part of this material should be copied or reproduced without permission
1 CHEMICAL REACTION KINETICS

pressure. Thus first of all we should


1.1 INTRODUCTION identify the constraints imposed by
Chemical reaction engineering subject was thermodynamics on the reaction.
developed with an aim of learning that
how to SELECT, SIZE and determine the 1.2 HOW A REACTION PROCEEDS
OPTIMAL CONDITIONS for a reactor
Let’s take the simple example of ammonia
whose purpose is to produce a specific
synthesis
chemical product. The word KINETICS is N2 + 3H2 2NH3
related to determining how fast or how
slow the reaction occurs. We are Suppose initially we have 10 moles of
interested in determining the speed of a nitrogen and hydrogen each and no moles
chemical reaction because it will affect the of product.
size (volume) of the reactor required. For
example if the speed of the reaction is high The coefficient of reactants and products
it will take less time (which is proportional in the reaction are called
to volume which we will understand later) STOICHIOMETRIC COEFFICIENT.
to reach to a specific value of conversion.
N 2 + 3H 2 → 2 NH 3
t = 0: 10 10 0
But it is not only the speed which matters.
The initial task in approaching the t = t1: 9 7 2
description of a chemically reacting
system is to understand the answer to the t = t2: 8 4 4
question that what changes are going to
occur. t = t3: 7 1 6
For example, nitrogen (N2) and hydrogen
(H2) are reacted over an iron catalyst t = t4 (7-1/3) 0 (6 + 2/3)
to give ammonia (NH3):
We can have the following conclusions
N2 + 3H2 2NH3
Generally at the exit of the reactor fifty  The total mass during a reaction
percent of nitrogen is converted. We can remains constant
expect that production of ammonia can be  Mass of a species and moles of a
increased significantly by adding a catalyst species change during the reaction
(lets say). But then THERMODYNAMICS  The reactant which gets consumed
tells us that we cannot improve the first is called the LIMITING
conversions significantly unless we REACTANT while the other
change the conditions of temperature &
reactant is called the EXCESS FAo & FA have the same meaning in
REACTANT. terms of flow rates.
 Most of the reaction does not go to
completion, they attain equilibrium  Conversion of all the reactants can
somewhere before the completion. be found, but because we do the
 All the stoichiometric calculations stoichiometric calculations on the
should be done on the basis of basis of the limiting reactant, hence
limiting reactant only. it is usually provided for the
limiting reactant.
1.2.1 Identifying the limiting  For a general reaction
reactant
aA + bB → cC + dD where we have
 Divide the initial moles of the
N Ao moles of A, N Bo moles of B, N Co
reactant with its stoichiometric
coefficient. moles of C & N Do moles of D
 The reactant for which the initially.
above value is less becomes the By using the reaction stoichiometry
limiting reactant. and assuming that A is the limiting
 If the reactants are present in reactant whose conversion is
stoichiometric proportion or known, we can write the following
stoichiometric ratio i.e., the equations
ratio of initial moles of
=N A N Ao (1 − X A )
reactants to their
corresponding stoichiometric b
N= N B0 − NA X A
coefficient are equal, then none B
a o
of the reactants are limiting. We c
can also say that both of them N=
C N C0 + N Ao X A
a
are limiting reactants.
d
N=
D N Do + N Ao X A
1.2.2 Conversion / Fractional a
conversion
 If conversion of one reactant is
 Conversion is only defined for known and the stoichiometry is
reactants and never for products. known, we can find all other moles.
 Conversion of reactant A is defined  The relation between conversions
as of two reactants is given by
bN Ao
N Ao − N A FAo − FA XB = XA
XA = or X A = aN B0
N Ao FAo
 The above equation suggests that if
 Here N Ao is the initial moles of A
the reactants are present in the
and N A is the moles of A left at the stoichiometric proportion then the
time when we want to calculate conversion of both the reactants
conversion. are same.
1.3 RATES OF A REACTION  Rate law has a general form of
rate = k{ f (T )only}* g{Concentration}
 The rate of a reaction can be
average rate or instantaneous rate. rate = k C αA CBβ
Average rate is defined between  k is called the rate constant or the
two instants of time while specific reaction rate and it is a
instantaneous rate is defined at a function of temperature only.
particular instant.  α & β are called order of the
 For the reaction aA + bB → cC + dD reaction with respect to A & B
−1dN respectively.
(− rA ) = A
Vdt  The overall order of the reaction is
−1dN denoted by n and n = α + β
(− rB ) = B
Vdt  The units of rate constant is given
1− n
1dN C  mol 
(rC ) = by  −1
 s if concentration is
Vdt  L 
1dN D expressed in mol/L and time in
(rD ) =
Vdt seconds.
 Since order of a reaction is
 The negative sign in the case of determined from the rate law,
reactants indicates that they are order of a reaction is an
being consumed. However the experimental quantity.
negative sign on the left side is only  Radioactive decay is an example of
for notation, it should never be first order reaction.
used for mathematical calculations.  The rate law of a reaction is
 In the above equations, V denote independent of the way in which
the volume of reaction mixture. It is the reaction is written.
usual practice to find the rates in
homogenous systems on the basis
of volume of reaction mixture. 1.4.1 ELEMENTARY & NON-
 The above rates are related as ELEMENTARY REACTIONS
(−rA ) (−rB ) (rC ) (rD )
= = =  The reactions which take place in a
a b c d
 Rates depends upon the single step only are called
concentration of the reacting elementary reactions.
species as well as the temperature.  For an elementary reaction, there is
Some other factors like catalyst can a direct correspondence between
also affect the rate of reaction. the order w.r.t a reactant and its
stoichiometric coefficient.
1.4 THE RATE LAW  For elementary reactions α = a &
 Rate law is an expression which is β = b. The overall order of the
obtained from experimental elementary reaction is hence equal
results. to a+b.
 Non-elementary reactions takes In VVRS, the volume of reaction mixture
place in more than one steps. One of changes as reaction proceeds. The volume
these steps is the slowest step of reaction mixture may increase or
which is the rate determining step decrease with conversion. In both cases it
also. is called variable volume reaction system.
 The rate law of a non-elementary
 All the liquid and aqueous phases
reaction is written from the rate
reactions, irrespective of
determining step.
stoichiometric coefficients are
 Just by looking at the reaction, we
considered as CVRS.
cannot say whether the reaction is
 If the reaction takes place in gas
elementary or not.
phase then we have to check
1.4.2 ORDER AND MOLECULARITY OF A whether the reaction is CVRS or
REACTION. VVRS
 For a gas phase reaction, if sum or
 Molecularity is only defined for
stoichiometric coefficients of
elementary reactions.
reactants and products is same,
 Molecularity of an elementary
then it is constant volume reaction
reaction is the number of molecules
system.
colliding to form products.
 For a gas phase reaction, if sum or
 The value of molecularity cannot be
stoichiometric coefficients of
zero, negative or fraction.
reactants and products is different,
 However order of a reaction can be
then it is variable volume reaction
zero, negative or fraction.
system.
 For an elementary reaction, order
of the reaction is equal to the 1.5.1 FRACTIONAL VOLUME CHANGE
molecularity of the reaction.
To give a qualitative meaning to change in
1.5 TYPES OF REACTION SYSTEM volume during the reaction we have
introduced ε , which is called fractional
In chemical reaction engineering we have
volume change. It is defined as
two types of reaction systems.
V −V
(1) Constant volume reaction system εA =
=
X 1=X
A A 0

(CVRS) VX A = 0

(2) Variable volume reaction system =V


XB 1= −V
Similarly we can define ε B =
XB 0
(VVRS) VX B = 0
In CVRS, the volume of reaction mixture
remains constant with conversion or with  ε is only defined for reactants
reaction time. This constant value of because its definition is based on
volume is equal to the initial volume of the the conversion of the reactants.
reaction mixture.  The value of ε indicates whether
the volume of reaction mixture is
constant or changing and hence it
tells about the volume change Species Ni@ XA=0 Ni@ XA=1
quantitatively. A 50 0
 If ε =0 there is no change in volume B 50 -25
and hence the system is CVRS. C 0 25
While if ε ≠ 0, there is change in D 0 25
volume and hence the system is TOTAL 100 25
VVRS.
 If ε is positive, there is an increase  Without knowing the actual
in volume with the reaction, while conversion we can calculate the
if ε is negative there is a decrease in value of ε .
volume of the reaction mixture
with reaction. Hence according to the definition of ε
 There is no need to find the value of 25 − 100
ε for aqueous and liquid phase εA = = −0.75
100
reaction irrespective of nature of
stoichiometric coefficients. Similarly, for the same reaction and initial
 The value of ε is always calculated feed
for complete conversion of the
Species Ni@ XB=0 Ni@ XB=1
reactant, irrespective of the actual
A 50 0
conversion. Hence we can say that B 50 50-
the value of ε is independent of (100/3)
actual conversion. C 0 50/3
Since at constant pressure and D 0 50/3
temperature, volume is directly TOTAL 100 50
proportional to the number of moles, we
can also define the value of ε as 50 − 100
εB = = −0.5
N −N 100
εA =
=
T|X 1=T|X
B B 0
where NT denotes
NT | X B = 0 With any of the values we can say that the
volume of reaction mixture decreases with
the total moles of reaction mixture.
time or with conversion.
Similarly ε B can be defined on the basis of
total moles. 1.5.3 RELATION BETWEEN VOLUME OF
REACTION MIXTURE AND ε A
1.5.2 CALCULATION OF ε
At constant temperature and pressure and
Let’s consider the following example of gas
assuming that the gaseous reaction
phase reaction.
mixture obeys ideal gas law, we have the
2 A + 3B → C + D following relations
N Ao = 50
V Vo (1 + ε A X A ) &=
= V Vo (1 + ε B X B )
N Bo = 50
For the same conditions we can write the =
C A C Ao (1 − X A )
above equations in the form of total moles
dC A = − C Ao dX A
also
dC A C Ao dX A
- NT NT o (1 + ε A X A ) &=
= NT NT o (1 + ε B X B ) (−rA ) =
− =
dt dt
The above equations also shows that total Constant volume systems are also called
volume of reaction mixture is a linear constant density systems.
function of conversion.
For variable volume systems because
However, if ideal gas law is not obeyed, volume of the system changes with the
then we have the following relation reaction and moles of species are also
PoT changing during the reaction there is no
V Vo (1 + ε A X A )
= direct relationship between moles and
PTo
concentrations. Hence different relations
 Hence from the above discussion it will be used. Variable volume problems
should be clear that the value of are also called variable density problems.
fractional volume change depends
NT NT o (1 + ε A X A )
=
upon the reaction stoichiometry
PoT
and the feed concentration. V Vo (1 + ε A X A )
=
PTo
εA ≠ 0
1.6 RELATIONS FOR CVRS & VVRS =
N A N Ao (1 − X A )
When there is no volume change during C Ao (1 − X A )
CA =
the reaction, all the relations which are (1 + ε A X A )
valid for moles of species or molar dX A
flowrates, are also valid for concentration (−rA ) =
C Ao
(1 + ε A X A )dt
of species.
But when there is a volume change during
the reactions, we should be careful while 1.7 KINETIC EQUATIONS FOR
writing different relations. CONSTANT VOLUME REACTION
For constant volume systems SYSTEM

NT = NT o The rate equations of the form


− dC A
V = Vo = kC An is called differential rate
dt
εA = 0 equation. This equation has to be
=
N A N Ao (1 − X A ) integrated to obtain integrated rate
equation or kinetic equation for different
orders of reactions for both constant
volume problems and variable volume
problems.
In the beginning we will start with
constant volume reaction systems of
different orders but with one type of
reactant only.
1.7.1 UNIMOLECULAR ZERO ORDER
REACTIONS (CVRS)

A  k
→ Pr oducts
1.7.2 FIRST ORDER REACTIONS
−dC A
=k
dt A  k
→ Pr oducts
−dC A = kdt −dC A
CA t
= kC A
dt
∫ −dC A k ∫ dt
=
−dC A
C Ao 0 = kdt
CA
C Ao − C A =
kt
−dC A
CA t
C Ao X A = kt

C Ao
CA
= k ∫ dt
0
 In a zero order reaction the rate
C A = C Ao e − kt
remains constant with conversion
or time. ln(1 − X A ) =
−kt
 A zero order reaction can go to
 It shows that rate of a first order
completion in finite time.
reaction decreases exponentially
 The time of completion of a zero
with time.
CA
order reaction is given by t = o  The conversion of a first order
k reaction increases exponentially
 The conversion obtained in a zero with time.
order reaction in a given time  A first order reaction never goes to
interval depends upon initial completion in finite time.
concentration.  The conversion obtained for a first
 The following plots can be obtained order reaction in a given time is
from above equations. independent of initial
concentration.
 Rate also decreases exponentially
for a first order reaction.
The following plots can be obtained from
the above equations.
For nth order CVRS we can write the
general equation of integrated rate
expression, which is given by
1 1
n −1
− n −1 =
(n − 1)kt ; n ≠ 1
CA C Ao

1.7.4 SECOND ORDER BIMOLECULAR


REACTION (CVRS)

A + B 
k
→ Pr oducts
−dC A
= kC ACB
dt
CBo
Let =M
C Ao

− dC A
= kC Ao (1 − X A )(CBo − C Ao X A )
dt
C Ao dX A
=kC Ao 2
(1 − X A )( M − X A )
dt
After integrating the above expression, we
get
1.7.3 SECOND ORDER REACTION
M − XA
A  k
→ Pr oducts ln =C Ao ( M − 1)kt ; M ≠ 1
M (1 − X A )
−dC A
= kC A2
dt  If M=1 i.e, the reactants are present
−dC A in stoichiometric proportion, then
= kdt it can be easily proved that the
C A2
integrated rate expression will be
−dC A
CA t
same as that of second order uni-

C Ao
C A2
= k ∫ dt
0 molecular reaction.
1

1
=
kt 1.7.5 IRREVERSIBLE PARALLEL
C A C A0 REACTIONS
XA Consider the reactant A reacting by two
= kC Ao t
1− X A paths where both paths are elementary.
These reactions are considered under the
 A second order reaction also never
category of multiple reactions. Because we
goes to completion in finite time.
have considered both reactions to be
 The conversion achieved in a given
elementary, both of them are of first order.
time interval depends upon the
initial concentration.
Also because of the stoichiometry we can 1.7.6 IRREVERSIBLE SERIES
say that REACTIONS
C Ao = C A + CR + CS A k1
→ R  k2
→S
−dC A
A 
k1
→R (−rA=
) = k1C A
dt
A  k2
→S dCR
=
rR = k1C A − k2CR
−dC A dt
−rA = = k1C A + k2C A = (k1 + k2 )C A
dt dCS
=
rS = k 2CR
dCR dt
=
rR = k1C A
dt
For the first reaction the relation is same
dCS
=
rS = k 2C A as of a first order reaction and is given by
dt
C A = C Ao e − kt
Integrating the above equation gives

C A = C Ao e − ( k1 + k2 )t If the value of CA is substituted in the


second differential equation, we get a
The concentration profile for this parallel linear differential equation in CR
react ion can be shown as dCR
+ k2CR =k1C Ao e − k1t
dt
The equation can be solved to give CR

 e − k1t e − k2t 
=CR C Ao k1  + 
 k2 − k1 k1 − k2 
Now by using the condition
C Ao = C A + CR + CS , we can calculate CS

 k2 e − k1t k1e − k2t 


CS =C Ao 1 + + 
For parallel reactions we can also write  k1 − k2 k2 − k1 
rR dCR k1
= = which on integrating gives The concentration profiles can be shown
rS dCS k2 as

CR − CRo k1
=
CS − CSo k2
Substituting the relations for equilibrium
conversion and M in the differential
equation, we obtain the following equation
dX A k1 ( M + 1)
= ( X Ae − X A ) and
dt M + X Ae
integrating,
XA M +1
− ln(1 − )= k1t which is the
X Ae M + X Ae
final integrated rate expression. Note that
this result is only valid for the given order
and given stoichiometry.
The concentration of R passes through a
maxima. The time at which concentration 1.7.8 AUTOCATALYTIC REACTIONS
of R is maximum can be obtained by
A reaction in which one of the products of
differentiating the expression of CR with
reaction acts as a catalyst is called an
respect to time and equating it to zero. autocatalytic reaction. The simplest such
k2 reaction is A + R → R + R .
ln dC
1 k1 −rA = − A = kC ACR
=
tmax = dt
klog mean k2 − k1
According to the reaction stoichiometry,
total moles of reactants and products
1.7.7 FIRST ORDER REVERSIBLE
remains constant.
REACTION
Co = C A + CR = C Ao + CRo
Let us consider the elementary first order dC
−rA = − A = kC A (Co − C A )
reversible reaction A ⇔ B . dt
dC A dX A dC A 1  dC dC A 
− = C Ao = k1C A − k2CB − = −  A+ =kdt
dt dt C A (Co − C A ) Co  C A Co − C A 
After integration it yields, assuming M is
= k1C Ao (1 − X A ) − k2 (CBo + C Ao X A ) the initial ratio of the reactants R to A.
M + XA
dX A k ln = C Ao ( M + 1)kt
= = k1C Ao [(1 − X A ) − 2 ( M + X A )] M (1 − X A )
dt k1
 For an autocatalytic reaction to
For reversible reactions, we can also proceed, there must be some
k amount of R present initially. As
define the equilibrium constant KC as 1 more R is formed, the rate
k2
increases. But, we know that as A is
and equilibrium conversion also getting consumed, the rate
M + X Ae C must fall to zero after some time.
X Ae = ,where M = Bo
1 − X Ae C Ao Hence the rate curve passes
through a maxima.
 Half-life of a first order reaction is
0.693
t1/2 = .
k
 Half-life of a first order reaction is
independent of initial
concentration while for a zero
order reaction, it depends upon
initial concentration.
 From the kinetics of nth order
system, it can be easily derived that
Half-life of nth order system is given
1.8 KINETIC EQUATIONS FOR (0.5)1− n − 1 1− n
VARIABLE DENSITY SYSTEMS by t1/2 = C Ao .
k (n − 1)
For zero order variable volume reaction
systems we can write  We can also say that half-life of a
C Ao dX A reaction is directly proportional to
= k Which on integration C1Ao− n . This relation can be used to
(1 + ε A X A )dt
determine the order of the reaction.
gives
Similar to half-life, we can also define
ln(1 + ε A X A )kt
= fractional life of a reactant. If X denotes the
εA C Ao fraction of A reacted, F denotes the
 For first order reactions, it can be fraction of A remaining. Hence X+F = 1.
easily proved that the kinetic Fractional life is the time taken by the
equation for variable volume reaction such that fraction F of reactant is
reaction system is same as of remaining. Hence fractional life for nth
constant volume reaction system. order reaction is given by
1− n
( F ) − 1 1− n
1.9 HALF -LIFE OF A REACTION tF = C Ao .
k (n − 1)
The time required for a reaction to reach  Fractional life of a reactant is
50% conversion is known as half-life of the directly proportional to ( F )1− n − 1 .
reaction. For ease of analysis, we will only  Above relation can also be used to
discuss constant volume reaction system determine the order of a reaction.
for calculation of half-life.
1.10 TEMPERATURE DEPENDENCY OF
The expression of half-life can be easily RATE
obtained by substituting, The rate constant present in the rate law
C A0 depends upon temperature only. The
=CA = or X A 0.5 in the kinetic effect of temperature on rate constant and
2
equations developed earlier. hence on the rate of reaction is given by
many theories. The most accepted theory
 Hence for a zero order reaction is the Arrhenius theory which relates k
C with T as
half-life is given by t1/2 = A0 . − Ea
2k k = ko e RT
where Ea is the activation energy. the increase for reaction 1 will be more
 Ko is called pre-exponential factor than increase for reaction 2.
and assumed to be independent of
temperature.
 The activation energy of a reaction
is always positive and is also
assumed to be independent of
temperature.
 The relation clearly shows that
with increase in temperature, rate
constant increases and hence rate
increases.
The Arrhenius relation can be written in a
different form for two values of
temperatures
k Ea  1 1 
=
ln 2  −  where k1 and k2 are
k1 R  T1 T2 
the rate constants at T1 & T2.

 Other theories were also given for


the reaction rates. According to
collision theory the rate constant is
related to temperature as
1 − Ea
k = koT 2 e RT
 According to transition state theory
the rate constant is given by
− Ea
k = koTe RT
 By default Arrhenius theory should
be used for solving numerical
problems unless specified.
On taking log on both sides of Arrhenius
E
=
theory, ln k ln ko − a . If we plot ln(k) vs
RT
E
1/T , it will be a straight line of slope a .
R
If we consider two different reactions of
activation energy Ea1 & Ea 2 such that
Ea1 > Ea 2 , then the plot shows that
reactions with higher activation energy
are more temperature sensitive. It means
that if temperature is increased, the rate
constant of both reactions will increase ,
GATE QUESTIONS

1. The reaction rate constants at two The overall order of the reaction is
different temperatures T1 and T2 are
(a)0.5 (b)1
related by
(c)1.5 (d)2
 k2  E  1 1 
(a)  k  R  T − T 
ln = (GATE 2001)
 1  2 1 

4. In the hydro dealkylation of toluene to


 k2  E  1 1  benzene, the following reactions occur
(b) =
ln   − 
 1
k R  T1 T2 
C7 H 8 + H 2 → C6 H 6 + CH 4
 k2  E  1 1  2C6 H 6 + → C12 H10 + H 2
(c) exp =   − 
 k1  R  T1 T2 
Toluene and hydrogen are fed to a
 k2  E  1 1  reactor in a molar ratio 1:5 80% of the
(d) exp =
   −  toluene gets converted and the
 k1  R  T2 T1  selectivity of benzene (defined as moles
of benzene formed /per moles of toluene
(GATE 2001)
converted) is 90% .the fractional
2. The conversion for a second order, conversion of hydrogen is
irreversible reaction (constant volume)
(a)0.16 (b)0.144
A k2
→ B , in batch mode is given by
(c)0.152 (d)0.136
1 k2C A0 t
(GATE 2002)
(a) 1 + k C t (b) 1 + k C t
5.The extent of a reaction is
2 A0 2 A0

2
(k2C A0 t ) k2C A0 t (a) Different for reactants and products
(c) 1 + k C t (d)
(1 + k C t )
2
2 A0
2 A0 (b) Dimensionless
(c) Dependent on the stoichiometric
(GATE 2001) coefficients
3. The following half-life data are (d) all of the above
available for the irreversible liquid phase
reaction, A → products: (GATE 2002)

Initial concentration Half-life 6. For an isothermal second order


(kmol/ m3 ) (min) aqueous phase reaction A → B , the ratio
2 2 of the time required for 90% conversion
8 1 to the time required for 45% conversion
is

© Copyright Reserved by Gateflix.in No part of this material should be copied or reproduced without permission
(a) 2 (b) 4 (d) (mol −1m −3 s −1 ), (mol −1/2 m3/2 s −1/2 )
(c) 11 (d) 22 (GATE 2005)
(GATE 2004) 10. Which is the correct statement from
7. The rate of ammonia synthesis for the the following statements on the
reaction N 2 + 3H 2  2 NH 3 is given by Arrhenius model of the rate constant
k = Ae − E / RT
=r 0.8 PN2 PH32 − 0.6 PNH
2
3
. If the reaction is
1 3 1 (a) A is always dimensionless
represented as, N 2 + H 2  NH 3 ,
2 2 2 (b) for two reaction 1 and 2, if A1 = A2
the rate of ammonia synthesis is
and E1 > E2, then k1 (t ) > k2 (t )
=
(a) r 0.8 P P − 0.6 PNH3 0.5 1.5
N2 H 2
(c) for a given reaction, the percentage
=
(b) r 0.8 PN2 P − 0.6 P 3 2 change of k with respect to temperature
H2 NH 3
is higher at lower temperatures
= (
(c) r 0.5 0.8 PN2 PH32 − 0.6 PNH
2
3
) (d) the percentage change of k with
respect to temperature is higher for
= (
(d) r 0.5 0.8 PN0.52 PH1.52 − 0.6 PNH3 ) higher A. (GATE 2005)

(GATE 2004) 11. For the liquid phase reaction S → P,


in a series of experiments in a batch
8. The rate expression for the gaseous reactor, the half-life (t1/2 ) was found to be
phase reaction CO + 2 H 2  CH 3OH is
inversely proportional to the square root
α
=
given by r k1 pCO pHβ 2 − k2 pCH
γ
3OH
of the initial concentration of A. The
order of the reaction is
α 1,=
(a) = β 1,=
γ 1
3
α 1,=
(b)= β 2,=
γ 1 (a) (b) 1
2
=
(c) α 1/=
3, β 2=
/ 3, γ 1 1 1
(c) + (d) −
2 2
α 1/ 2,=
(d) = β 1,=
γ 1/ 2
(GATE 2005)
(GATE 2004)
12. For the reaction 2 R + S → T , the
9. The rate expression for the reaction of
rates of formation rR , rS and rT of the
k C2
A is given by −rA = 1 A 1/2 . The units of substances R, S and T respectively, are
1 + k2C A
related by
k1 and k2 are respectively
(a) 2rR= rS= rT (b) 2rR = rS = −rT
−1 3 −1 −1/2
(a) (mol m s ), (mol 3/2
m )
(c) =
rR 2=
rS 2rT (d) rR = 2rS = −2rT
−1 3 −1
(b) (mol m s ), (mol m ) 1/2 3/2

(GATE 2005)
−3 −1 −3/2 −1
(c) (molm s ), (mol m 1/2
s )

© Copyright Reserved by Gateflix.in No part of this material should be copied or reproduced without permission
13 .The rate, at which an antiviral drug 16 .The half-life of an nth order reaction
acts, increases with its concentration in in a batch reactor depends
the blood C, according to the equation
(a) only on the rate constant
kC
r= (b) on the rate constant and the order of
C50 + C the reaction
Where C50 is the concentration at which (c) On the rate constant and the initial
the rate is 50% of the maximum rate K. reactant concentration
Often the concentration C90 of the (d) On the rate constant, initial
maximum, is measured instead of C50 . concentration and the order of the
The rate equation then becomes reaction (GATE 2012)

1.8kC kC 17. The gas phase decomposition of


(a) r = (b) r = azomethane to give ethane nitrogen
C90 + C  C90 
 +C takes place according to the following
 9 
sequence of elementary reaction.
kC 0.9 kC
(c) r = (d) r = ∗
C90 C90 ( CH 3 )2 N 2 + ( CH 3 ) N 2 → K1 ( CH 3 )2 N 2 + ( CH 3 )2 N 2 

(GATE 2006) ( CH 3 ) N 2  + ( CH 3 )2 N 2 → K 2 ( CH 3 )2 N 2 + ( CH 3 )2 N 2

14.For a homogenous reaction system, ( CH 3 )2 N 2  → K3C2 H 6 + N 2
where
using the pseudo-state-approximation
Ci - is the concentration of i at time t ∗
for ( CH 3 )2 N 2  , the order with respect
N i is the number of moles of i at time t
to azomethane in the rate expression for
V is the reaction volume at time t the formation of ethane, in the limit of
t is the reaction time, high concentrations of azomethane, is
The rate of reaction for species j is
defined as (a) 0 (b) 1
dC j  dC  (c) 3 (d)2
(a) (b) −  j 
dt  dt 
(GATE 2013)
1 dN j  1 dN j 
(c) (d) −   18. A homogeneous reaction ( ( R → P)
v dt  v dt 
occurs in a batch reactor. The conversion
(GATE 2009)
of the reactant R is 67% after 10 minutes
15. The half-life of a first order liquid and 80% after 20 minutes. The equation
phase reaction in 30 sec. Then the rate for this reaction is
constant in min −1 is
(a) −rR =
k (b) −rR =
kCR2
(a) 0.0231 (b) 0.602
(c) −rR =
kCR3 (d) −rR =
kCR0.5
(c) 1.386 (d) 2.0
(GATE 2014)
(GATE 2009)

© Copyright Reserved by Gateflix.in No part of this material should be copied or reproduced without permission
19.An irreversible, homogeneous
reaction A → Products, has the rate
expression:

2C A2 + 0.1C A
Rate = , where C A the
1 + 50C A
concentration of A is C A varies in the
range 0.5-50 mol/m3 .For very high
concentration of A, the reaction order
tends to: Select the reaction scheme that correctly
represents the above plot. The numbers
(a) 0 (b) 1 in the reaction schemes shown below,
represent the first order rate constants
(c) 1.5 (d)2
in unit of s −1 .
(GATE 2015)
(GATE 2016)
20.For which reaction order, the half-life
of the full lifetime (time for 100% (a) (b)
conversion) of the reactant?
(a) Zero order (b) Half order
(c) First order (d) Second order
(GATE 2015)
21. Hydrogen iodide decomposes (c) (d)
through the reaction 2 HI  H 2 + I 2 . the
value of the universal gas constant R is
8.314 J mol −1 K −1. the activation energy
for the forward reaction is 184000J mol −1
. ratio (rounded off to the first decimal
place) of the forward reaction rate at 600 23. For a non-catalytic homogeneous
K to that at 550 K is______ reaction A → B, the rate expression at
10C A
(GATE 2016) 300 K is −rA (molm −3 s −1 ) = ,where
1 + 5C A
22. The variations of the concentrations C A is the concentration of A (in
(C A, CR and CS ) for three species (A,R
mol / m3 ). Theoretically, the upper limit
and S) with time, in an isothermal
for the magnitude of the reaction rate
homogeneous batch reactor are shown
in the figure below. (−rA in molm3 s −1 , rounded off to the first
decimal place) at 300 K is_______
(GATE 2016)

© Copyright Reserved by Gateflix.in No part of this material should be copied or reproduced without permission
24. For a chemical reaction. The ratio of
rate constant at 500 K to that at 400 K is
2.5. given R=8.314 J mol −1 K −1 ,the value of
activation energy (in kJ/mol) is
(a) 10.5 (b)12.0
(c)15.2 (d)18.4
(GATE 2018)

© Copyright Reserved by Gateflix.in No part of this material should be copied or reproduced without permission
ANSWER KEYS
1 2 3 4 5 6 7 8 9 10
(B) (B) (C) (C) (D) (C) (B) (A) (A) (C)
11 12 13 14 15 16 17 18 19 20
(A) (D) (B) (C) (C) (D) (B) (B) (B) (A)
21 22 23 24
(28.5) (C) (2) (C)
EXPLANATIONS

1.Solution:- (b) X A= (1 − X A )k2C A0t


k2C A 0t
Assuming Arrhenius law XA =
1 + k 2C A 0t
k = Ae − E / RT
3. Solution:- (c)
E 1 1 
k1  − 
dC A
=e R  T1 T2 
For nth order, − =
kC An ...(1)
k2 dt
Or
CA dC A t
k  E 1 1  −∫ ∫
= kdt
ln  1  =−  −  C Ao CAn 0

Or  k2  R  T1 T2 
C1A− n − C1A−o n =k (n − 1)t
 k1  E  1 1 
ln =   −  C Ao
 k2  R  T1 T2  For half-life, t = t1/2 and C A =
Or 2

2.Solution:- (b) 2n −1 − 1 1− n
t1/2 = C A ...(2)
k (n − 1) o
The rate equation for second order
reaction can be written as Putting values from given data,
(2n −1 − 1) 1− n
−dC A0 =2 × 2 ....(3)
= k2C A2 k (n − 1)
dt
(2n −1 − 1) 1− n
−dC
CA t
=
And 1 × 8 ....(4)
∫C CA 2A = ∫0 k2 dt k (n − 1)
A0

Dividing Equation (4) by Equation (3),


1 1
− =
k2t 1− n
C A C A0 1 8
= 
1 1 2 2
− =
k2t
C A (1 − X A ) C A0 1
= 41− n ⇒ n= 1.5
XA 2
= k 2t
C A0 (1 − X A )
4. Solution:- (c) 1  1 
Or t=  
C7 H 8 + H 2 → C6 H 6 + CH 4 kC Ao  1− X A 

Initial moles a 5a 0 0 Time required for 90% conversion,


1  0.9  9
After 80% a(1-0.8) 5a-0.8a 0.8a 0.8 a=t90 =  
kC Ao  1 − 09  kC Ao
And 2C6 H 6  C12 H10 + H 2
Time required for 45% conversion,
0.8a-2b b 45 1
t45 = .
Moles Of benzene formed 55 kC Ao
Given, =0.9
Moles of toluene converted t 55
Thus, t45 =90 =×
9 =11
0.8a − 2b t45 45
= 0.9
0.8a
7.Solution:(b)
Or b = 0.04a
N 2 + 3H 2  2 NH 3
Fractional conversion of hydrogen
=r 0.8 pN2 p 3 NH 3 − 0.6 p 2 NH 3
0.8a − b 0.8a − 0.04a
= = = 0.152 The rate of the reaction does not change
5a 5a
even if we change the stoichiometric
5.Solution:(d) coefficient by a certain factor.
All options are true 1 3
∴ For , N 2 + H 2  NH 3
6. Solution:- (c) 2 2

A→ B The rate of reaction remains same.

For second order reaction, 8.Solution :(b)

−dC A CO + 2 H 2  CH 3OH
( −rA=) = kC A2
dt The rate of reaction
= C A C A0 (1 − X A ) α
=r k1 pCO pHβ 2 − k2 pCH
γ
3OH
Since, dX A
∴ C A0 = kC A20 (1 − X A ) 2 For this gas phase reaction α , β and γ
dt
XA
should be in proportion to their respective
t
dX A stoichiometric coefficients.
∫ (1 − X A ) 2
= kC Ao ∫ dt
0 0
 Thus, α : β : γ = 1: 2 :1
XA
 1  ∴α : β : γ ≠ 1:1:1
  = kC Ao t
1 − X A  0
9. Solution:- (a) 11.Solution:- (a)

k1C A2 A→ P
Given, −rA =
1 + k2C1/2A For nth order reaction, the rate equation is

Or (−rA ) + (−rA )k2C A =k1C A − dC A


1/2 2
= kC An
dt
Each term in this equation will have same
Or C1A− n − C1A−0n =(n − 1)t
unit, i.e, (−rA )
mol At half-life, C A = C A0/2
The unit of (−rA ) =
m3 − s
2n −1 − 1 1− n
∴ t1/2 = C A0
∴ unit of k1C A2 =
mol k (n − 1)
m3 − s
Now, check for all options given,
2
 mol   m 
3

Or k1 =  m3 − s   mol  n=
3
  2
−1 3 −1 1
Or k1 = mol m s t1/2 α C A−1/2
0 α
C A0
Now, unit of (−rA ) k2C A = unit of (−rA )
1/2

3
∴ n = is correct answer.
1/2 2
 mol 
Or k2  3  =1
 m  12.Solution: (d)
−1/2 3/2 2R + S → T
Or k2 = mol m
The rates can be written for different
10.Solution:- (c)
substance as rR = 2rS = −2rT
The rate constant from Arrhenius model
kC
− E / RT 13. Solution:- (b)Given, r = ...(1)
k = Ae C50 + C
Option (a) Also, rmax = k and r90 = 0.9k
− E / RT
e is dimensionless.
kC90
Or = 0.9k
∴ Units of K and A are same. C50 + C90
∴ A is not dimensionless. =
or kC90 0.9k (C50 + C90 )
Option(b) If A1 = A2
C90 = 9C50 .....(2)
1
k1 ( E1 − E2 )
= e RT From Eqs. (1) and (2)
k2
kC 17. Solution:- (b)
r=
C90
+C [C2 H 6 ] depends on [ (CH 3 ) 2 N 2 ] which in

9
turn depends on [ (CH 3 ) 2 N 2 ] . the ratio of
14.Solution:- (c)
all concentration is one. The order of
1 dN
Rate of  reaction = . j reaction w.r.t (CH 3 ) 2 N 2 is 1
V dt
Option(b) is also rate of reaction but for 18. Solution:- (b)
liquid phase reaction only.
For ideal batch reaction
15.Solution: (c) CA
dC A
CA
dC A
=A ∫= ∫
For first order reaction CA0
−γ a CA0
KC An

0.693
t1/2
k =
C A C A0 (1 − X A )
=
k
0.693
=, 30
= s
1 dC A = −C A0 dX A
t1/2 2
C1A−0n 4 dX A
X
dX A
−C A0 ∫
K ∫0 (1 − X A ) n
= 1.386 min −1 t= =

KC An 0 (1 − X A ) n
16. Solution:- (d) C1− n  (1 − X A )1− n 
XA

t = A0  
dC K  1− n 
For nth order reaction, − =
kC n
dt C1− n  (1 − X A )1− n − 1 
t = A0  
C
dC
t K  1− n 
Or ∫C C n = ∫ −kdt
For reaction R → P;
o 0

1  1 1  X A =0.67, t =10 minutes


Or  n −1 − n −1  =
−kt
1− n C C0  X A = 0.08, t = 20minutes
C0
=
For half-life, =
t t1/2 ,C  0.331− n − 1 
2 10 = C1A−0n   ____(1)
 1− n 
1 1
. (2n −1 − 1) =
kt1/2
n −1
C0 (n − 1) C1A−0n  0.21− n − 1 
20 =
K  1 − n 
______(2)
It can be seen that the half-life depends on
rate constant(k), initial concentration Dividing equation (1) by equation (2) =
( C0 ) and order of the reaction (n). 1 0.331− n − 1
=
2 0.21− n − 1

Solving we get n =2 ⇒ So, −γ R =KCR


2
19.Solution: (b) 21.Solution:

2C A2 + 0.1C A  k2  E  1 1 
Rate = ln =   − 
1 + 50C A  k1  R  T1 T2 
0.5 < C A < 50(mol / m3 ) k2
= 28.5
k1
For very high value of C A (say (50mol / m3 )
22. Solution:- (c)
0.1C A << 2C 2
A
23.Solution:
and 50C A >> 1
10C
2C A2 1 −ra = A
so,=
that = CA 1 + 5C A
50C A 25
10C A
So reaction order is one @ highC A − r=
A = 2
5C A
20.Solution: (a)
10C A
For zero order reaction @ lowC A − r=
A = 10C A
1

= 10C A ; 0 [lowC A ]
dC A
kC A0 = k
dt
Upper limit −rA =
2
Where k=rate constant
24. Solution:- (c)
C A0 − C A =
kt
k2 E  1 1 
For, full life time C A =0 =
ln  − 
k1 R  T1 T2 
C A0 Ea  1 1 
t= ln ( 2.5 )
= −
K 8.314  400 500 

And for half life E a = 15.24 kj/mol

C A0 C
CA = , So t 1 = A0
2 2 2k

So, t = 2t 1
2
2 DESIGN OF ISOTHERMAL IDEAL SINGLE
REACTORS

2.1 INTRODUCTION 2.2 IDEAL BATCH REACTORS


So far we were dealing with kinetics, i.e.,  The batch reactors are used in
how much time reaction will take to reach those cases where we have to
a particular conversion. But the question is produce less amount of products.
which reactor is suitable for the operation.  In batch reactors there is no
The reaction should be carried out in batch continuous inlet of reactants and
mode or continuous mode or semi-batch no continuous outlet of products.
mode. In this chapter we will deal with  All the reactants are charged at
choice and design of reactors. Design once and they are allowed to react
means sizing the reactor or finding the for some time. Once desired
volume of reactor required for a given conversion is achieved, the
conversion .To find the volume of the products are discharged at once.
reactor required, design equation or  Since no continuous input or
performance equation will be derived. output is there, the IN and OUT
terms of material balance are
The performance equation of a reactor can
ZERO.
be derived by writing material balance on
 In batch reactors we talk in terms
any of the reactants or products. Because
of amount of substance, not in
we are dealing with isothermal reactors in
terms of flow rates of species.
this chapter, so energy balance is not
 A batch reactor is considered to be
required. The general equation of material
uniformly or perfectly mixed but
balance on a species A is given by
the conditions inside the reactor
(moles Ain) − (moles Aout ) changes with time and hence it is an
−(moles Areacted ) unsteady-state reactor.
+(moles A generated ) =( Accumulation of moles of A)  However the total mass inside the
reactor remains constant, the
In this chapter we will deal with three species mass changes with time.
types of reactors Similarly other properties in the
reactor also change with time.
(i) Batch reactors & semi-batch reactors
Hence the material balance becomes,
(ii) Continuous Stirred Tank Reactors
dN A
(iii) Plug Flow Reactors 0 + 0 – (−rA ) V + 0 = where V is the
dt
volume of reaction mixture. Integrating,
−dN A  Hence all the previously derived
dt =
(−rA )V equations can be used for the
reactions taking place in a batch
−dN A
t XA

∫ dt = ∫ VkC
0 0
n
Ao (1 − X A ) n
reactor.
2.2.1 PERFORMANCE CURVES OF
The above integration has to be solved and BATCH REACTORS
it relates the time required to achieve a
Performance curves are the plots between
given conversion in the batch reactor.
1 1
For constant volume reactions and for Vs X A or Vs C A .
variable volume reaction systems (batch −rA −rA
reactor) the value of V has to be
substituted and the integral has to be
solved.
For constant volume batch reactors the
above integral reduces to
XA
t dX A
C Ao
= ∫ (−r )
0 A

XA
dX A
t = C Ao ∫ kC
0
n
Ao (1 − X A ) n

For variable volume batch reactors, the


volume has to be kept inside the integral

−dN A
dt =
(−rA )V
XA
dX A
t = N Ao ∫ (−r )V
0 A

 A very important conclusion for


batch reactors is the design or
performance equation of batch
reactors is same as the kinetic
equations.
 It means that when we integrate
the above equations for different
orders and different systems (CVRS
& VVRS) , we will obtain the same
equations that we have obtained
for kinetic equations.
 For batch reactors , the shaded area  Space time and space velocity are
( area under the curve) gives only defined for flow reactors and
indication of how much time not for batch reactors.
should be given to the reactants to  The average residence time t can
achieve desired conversion. also be defined for flow reactors. It
can be understood as the ratio of
2.3 CONTINUOUS REACTORS
volume of the reactor to the
 The reactors where there is a average volumetric flowrate.
continuous inlet stream of  During gas phase reactions there
reactants and product is also may be increase or decrease in total
withdrawn continuously are moles and due to this the
continuous reactors. volumetric flowrate also increases
 In continuous reactors we can talk and decreases which affects the
in terms of flow rates. average residence time.
 Flow rates can be molar flow rates,  Hence for constant volume systems
mass flow rates, or volumetric where volumetric flowrate at the
flowrates. inlet and inside the reactor are
 Continuous reactors can also be same, the value of space time and
operated at unsteady state, but we average residence time should be
always do the analysis of these same.
reactors at steady state only.  For variable volume systems, the
However the reactors may or may volumetric flow rates inside the
not be uniform. reactors may be less or more than
 For continuous reactors we define that of the inlet, therefore space
two terms, space time τ and time and residence time are
1 different.
space velocity .
τ  The above discussion suggests that
 Space time is the time required to for those gas phase reactions
process one reactor volume. The where there is increase in moles
space time has the units of time. with reactions, average residence
V time should be less than space time
 τ = R where VR is the reactor and vice versa.
νo
volume and v0 is the inlet
volumetric flowrate. Space time is
always calculated at the inlet 2.3.1 IDEAL MIXED FLOW REACTORS
conditions and it is independent of The mixed flow reactors are also known as
what is happening inside the continuous stirred tank reactors (CSTR),
reactor. and Constant flow stirred tank reactors
 The reciprocal of space time is (CFSTR). The ideal mixed flow reactors
called space velocity and defined as has the following characteristics
the number of reactor volumes
processed per unit time.
 The contents of the reactor are flow rates are independent of change in
perfectly mixed and hence they are volume, hence the equation can be used
uniform in composition. for both constant and variable volume
 Analysis of MFR is done at steady systems.
state.
However if we substitute the value of XA
 The composition and properties of
exit stream of the MFR is same as CA
X A = 1−
that of the properties of the C Ao
mixture inside the reactor.
VM C A0 − C A
 In a MFR there is a step change in =
υ − rA
properties of the inlet stream. Its
concentration decreases suddenly C A0 − C A
τM =
as it enters the reactor. Similarly − rA
there is step change in properties
This equation can only be applied to
like temperature, volumetric
constant volume systems. We can observe
flowrates etc.
from the above equations that conversion
For a reaction A → Pr oducts , the material achieved is proportional to the volume of
balance becomes the reactor. Higher the volume more is the
conversion if rest everything remains
same.
2.3.2 PERFORMANCE CURVES
VM X
Using the general equation = A
FAo −rA

FAo − FA − (−rA )V − 0 =0
FAo − FA0 (1 − X A ) − (−rA )V =0
FAo X A = (−rA )V
VM X
= A
FAo − rA
We do not consider the area of the curve
In the above equation VM denotes the for a MFR. We consider the area of the
volume of the MFR required to achieve a rectangle. This rectangle is completed
conversion XA. according to the final conversion achieved
Because the material balance was written in the reactor. For constant volume
on the basis of molar flow rates and molar systems the performance curve becomes
 Hence in an ideal PFR there will be
concentration gradient in the axial
direction while no concentration
gradient in the radial direction.
 Each plug of liquid enters the
reactor and spends same time
inside the reactor.
 At a given time, there are many
such plugs inside the PFR. Each
plug is having a different
concentration of reactants because
 While solving problems on they have spent different time
mixed flow reactors the inside the reactor. Hence the plug
value of rate has to be flow reactor is non-uniform.
substituted in the design  But if we consider a fixed location
equation. inside the reactor, then at that
 The rate in the design location concentration of any plug
equation should always be will be same.
calculated according to the  Any property, concentration or
exit conversion. temperature or conversion
VM XA changes gradually in a PFR.
For CVRS =
FAo kC Ao (1 − X A ) n
n
Since the PFR is non-uniform, we cannot
select the entire reactor as the control
VM X (1 + ε A X A ) n volume for writing the material balance
For VVRS = An
FAo kC Ao (1 − X A ) n equation. So we will select a thin strip of
reactor of volume dV to write the balance
2.3.4 IDEAL PLUG FLOW REACTORS
and then integrate it over the entire
Plug flow reactors are also called tubular volume of the reactor
reactors or unmixed reactors or piston
flow reactor.

 In an ideal PFR there is no axial


mixing. The liquid elements enter
as a plug and the successive plugs
are not mixed with each other.
 There is infinite radial mixing
inside the PFR.
FA − ( FA + dFA ) − (−rA )dV + 0 =0
Q dFA = − FAo dX A
dV dX A
∴ =
FAo − rA
VP XA
dV dX A

0
FAo
= ∫0
−rA
dX A τ P
XA XA
VP dX A
=
FAo ∫
0
= =
−rA C A0 ∫
0
−rA

The above equation can be solved for


different orders. Hence for constant
volume reaction systems and first order Because of the integral in the design
systems, equation, the area under the curve is
calculated. For constant volume reaction
τP XlA
dX A systems, the performance curve takes the
C Ao
= ∫
0
kC Ao (1 − X A ) following form.
XA
dX A
kτ P = ∫ 1− X
0 A

kτ P =
− ln(1 − X A )

 We observe that the design


equation of PFR for constant
volume reaction systems is same as
the kinetic equation if we replace t
in kinetic equation with τ P .
 But for variable volume reaction
systems there is no direct 2.3.6 SPACE TIME AND MEAN
correspondence between kinetic RESIDENCE TIME
equation and design equation.
 Hence for solving problems on For both CSTR and PFR, space time is
constant volume reaction systems calculated by dividing the volume of the
in plug flow reactors, there is no reactor by inlet volumetric flowrate. For
need to solve the integral and the constant volume reactions the volumetric
kinetic equations can be used flowrate does not change inside the
directly. reactor and hence mean residence time
remains same as that of space time for
2.3.5 PERFORMANCE CURVES both the reactors.
Using the equations developed we can plot For variable volume reaction in a CSTR,
the following curves there is sudden expansion or contraction.
The volumetric flowrate inside the MFR is
different from the inlet volumetric For positive order reactions,
flowrate, but remains at a constant value
inside the reactor. This value of flowrate
inside the reactor is also same as that of
exit volumetric flow rate. Hence for this
case average residence time can be found
by dividing the reactor volume by exit
volumetric flowrate.
For variable volume reaction in a PFR the
volumetric flowrate within the reactor is
different than the inlet volumetric
flowrate. But the flowrates inside the PFR
are not constant. It decreases or increases
gradually and hence average residence It is clear that the area under the curve is
time in a PFR can be derived as less than the area of the rectangle for a
XA given conversion.
dX A
t = C Ao ∫ (−r )(1 + ε
0 A A X A)  Hence for positive order reactions
and for a fixed conversion, the
If there is increase in number of moles volume required by a CSTR is more
during a gas phase reaction, volumetric than the volume required by a PFR.
flowrate will increase with conversion and  For a positive order reactions and
hence average residence time will be less for fixed volume of reactors, a PFR
than the space time and vice-versa. always gives higher conversion
than a CSTR.
This statement is only valid for gas phase
 For positive order reactions, a PFR
reactions when ideal gas assumption is
should be selected.
valid.
1
2.3.7 SELECTION OF REACTORS For zero order reactions, vs X A is
−rA
Here we have to make a selection between
ideal CSTR and ideal PFR. The selection
criteria is to minimize the volume of
reactor for a given conversion or to
maximize the conversion for a given
reactor volume. The type of reactor
suitable for a reaction is decided according
to order of the reaction. That reactor is
selected for which the area of the
performance curve (area under the curve
for PFR & area of the rectangle for CSTR) is We observe that for a zero order reaction,
minimum. the area under the curve as well as the area
of rectangle are same. Hence for a given
conversion the volume of PFR & CSTR
required will be same.
For negative order reactions,

Area for CSTR is less than area for PFR.


 Therefore for negative order
reactions & for a given conversion,
volume of PFR required is more
than the volume of CSTR required.
 For a negative order reaction, and
fixed volume of reactors, CSTR will
give higher conversions than PFR.
 In this case CSTR should be
preferred.
We can summarize the selection of
reactors as
n>0 select PFR
n = 0 type of reactor doesn’t matter
n < 0 select CSTR
where n is the order of reaction.
GATE QUESTIONS

1.The first order series reaction X A (1 + X A )


(b) kτ =
A 
k1
→ B 
k2
→ C is conducted in a batch (1 − X A )
reactor. The initial concentration of A, B
and C ( C A0 , CB 0 , CC 0 respectively) are all XA
(c) kτ =
(1 − X A ) 2
non-zero. The variation of CB with
reaction time will not show a maximum, if X A / (1 + X A ) 2
(d) kτ C Ao =
(a) K 2CB 0 > K1C A0 (b) K1C A0 > K 2CB 0 (1 − X A ) 2

(c) CB 0 > C A0 (d) C A0 > CB 0 (GATE 2003)


4. A pollutant P degrades according to first
(GATE 2001) order kinetics. An aqueous stream
2. The first-order, gas phase reaction containing P at 2 kmol/ m3 and volumetric
A 
k1
→ 2 B is conducted isothermally in flow the 1 m3 /h requires a mixed flow
batch mode. The rate of change of reactor of volume V to bring down the
conversion with time is given by pollutant level to 0.5 kmol/ m3 . The inlet
concentration of the pollutant is now
dX A
(a) =
k1 (1 − X A ) 2 (1 + 2 X A ) doubled and the volumetric flow rate is
dt
tripled. If the pollutant level is to be
dX A brought down to the same level of 0.5
(b) =k1 (1 − X A )(1 + 0.5 X A )
dt kmol/ m3 , the volume of the mixed flow
reactor should be increased by a factor of
dX A
(c) = k1 (1 − X A )
dt (a) 7 (b) 6

dX A k1 (1 − X A ) (c) 3 (d) 7/3


(d) =
dt (1 + X A ) (GATE 2004)
(GATE 2001) 5. An isothermal aqueous phase reversible
reaction P ↔ R is to be carried out in a
3. An elementary liquid phase
mixed flow reactor. The reaction rate in
decomposition reaction A → 2 B is to be
carried out in a CSTR .the design equation (kmol/ m3 -h) is given by r=0.5 CP -0.125
is CR . A stream containing only P enters the

XA reactor. The residence time required (in


(a) kτ = hour) for 40% conversion of P is
(1 − X A )
(a) 0.80 (b) 1.33 (1 − X )
(a) C A = 1.2C A0
(1 − 0.33 X )
(c) 1.60 (d) 2.67
(1 − X )
(GATE 2004) (b) C A = 1.2C A0
(1 − 0.5 X )
6. A second order liquid phase reaction (1 − X )
(c) C A = 0.83C A0
A → B is carried out in a mixed flow (1 − 0.33 X )
reactor operated in semi-batch mode (no (1 − X )
exit stream). The reactant A at (d) C A = 0.83C A0
(1 − 0.5 X )
concentration C AF is fed to the reaction at
(GATE 2004)
a volumetric flow rate of F. The volume of
the reacting mixture is V and the density of 8. The rate of the liquid phase reversible
the liquid mixture is constant. The mass reaction A  2 B in (kmol/ m3 ) at 298 0 K ,
balance for A is is =
−rA 0.02C A − 0.01CB , where the
d (VC A ) concentrations C A and CB are expressed
(a) =
− F (C AF − C A ) − KC A2V
dt in (kmol/ m3 ). What is the maximum
limiting conversion of A achievable in an
d (VC A )
(b) = F (C AF − C A ) − KC A2V isothermal CSTR at 298K, assuming pure A
dt is fed at the in inlet?
d (VC A ) (a) 1 (b) 2/3
(c) =
− FC A − KC A2V
dt
(c) 1/2 (d) 1/3
d (VC A )
(d) = FC AF − KC A2V (GATE 2005)
dt
9. An irreversible gas phase reaction
(GATE 2004)
A → 5 B is conducted in an isothermal
7. The following gas phase reaction is batch reactor at constant pressure in
taking place in a plug flow reactor, the presence of an inert. The feed contains
1 no B. If the volume of the gas at complete
A+ B → C
2 conversion must not exceed three time the
initial volume, the minimum mol % of the
A stoichiometric mixture of A and B at 300 inert in the feed must be
K is fed to the rector. At 1 m along the
length of reactor, the temperature is 360 K. (a) zero (b) 20
The pressure drop is negligible and an (c) 33 (d) 50
ideal gas behaviour can be assumed.
Identify the correct expression relating the (GATE 2006)
concentration of A at the inlet (C A0 ) , 10. A first order reversible reaction
concentration of A at exit (C A ) and the A  B occurs in a batch reactor. The
corresponding conversion of A( X ) . exponential decay of the concentration of
A has the time constant
1 1 A→ B→C A→ R
(a) (b)
k1 k2
Is to be carried out in the an isothermal
1 1 CSTR. The rate laws are given by
(c) (d)
k1 − k2 k1 + k2 rR = k ' C A
=
rB kC A − kCB
(GATE 2006)
11. A well-stirred reaction vessel is Feed is pure A. The space time of the CSTR
operated as a semi batch reactor in which which results in the maximum exit
it is proposed to conduct a liquid phase concentration of B is given by
first order reaction of the type A → B . The 1 1
reactor is fed with the reactant A at a (a) (b)
kk ' k' (k + k' )
constant rate of 1 L/min having feed
concentration equal to 1 mol/L. The
1 1
reactor is initially empty. Given k=1 min −1 , (c) (d)
the conversion of reactant A based on (k + k ) '
k (k + k' )
moles of A fed at t=2 min is
(GATE 2008)
(a) 0.136 (b) 0.43
14. The liquid phase reaction
(c) 0.57 (d) 0.86 A → Products is governed by the kinetics,
(GATE 2007) (−rA ) = A . if the reaction undergoes
kC1/2
75% conversion of A in 10 min in an
12. The gas phase reaction A + 3B → 2C is
isothermal batch reactor, the time (in min)
conducted in a PFR at constant
for complete conversion of A is
temperature and pressure, the PFR
achieves a conversion of 20% of A. The (a) 40/3 (b) 20
feed is a mixture of A, B and an inert I. It is
(c) 30 (d) infinite
found that the concentration of A remains
the same throughout the reactor. (GATE 2008)
Which one of the following ratios of inlet 16. An isothermal liquid phase zero order
molar rate ( FA , in : FB , in : FI , in) is reaction A →= B(k 0.5mol / m3 − s ) is
consistent with this observation? Assume carried out in a batch reactor. The initial
the reaction mixture is an ideal gas concentration of A is 2mol / m3 . At 3
mixture. seconds from the start of the reaction, the
(a) 2:3:0 (b) 2:2:1 concentration of A in mol / m3 is
3 3
(c)3:2:1 (d)1:2:1 (a) 0.8 mol / m (b) 0.0 mol / m

(GATE 2008) (c) 0.5 mol / m


3
(d) 0.1 mol / m
3

13. The elementary liquid phase series (GATE 2013)


parallel reaction scheme
17. In order to achieve the same 19. The liquid phase reversible reaction
conversion under identical reaction A  B is carried out in an isothermal
conditions and feed flow rate for a non- CSTR operating under steady state
autocatalytic reaction of positive order, conditions. The inlet stream does not
the volume of an ideal CSTR is contain B and the concentration of A in the
inlet stream is 10mol/lit. The
(a) Always greater than that of an ideal
concentrations of A at the reactor exit, for
PFR
residence times of 1 s and 5 s are 8 mol/lit.
(b) Always smaller than that of an ideal And 5mol/lit, respectively. Assume the
PFR forward and backward reactions are
elementary following the first order rate
(c) Same as that of an ideal PFR
law. Also assume that the system has
(d) Smaller than that of an ideal PFR only constant molar density. The rate constant
for first order reaction of the forward reaction (in s −1 , rounded
(GATE 2014) off to the third decimal place) is _________

18. An isothermal steady state mixed flow (GATE 2016)


reactor (CSTR) of 1 m3 volume is used to 20. The following liquid phase second
carry out the first order liquid-phase order reaction is carried out in an
reaction A → products. Fresh feed at a isothermal CSTR at steady state
volumetric flow rate of Q containing
reactant A at a concentration C Ao mixes A → R (−rA ) =
0.005C A2 mol / m 2 hr

with the recycle stream at a volumetric Where CA is the concentration of reactant


flow rate RQ as shown in the figure below.
in the CSTR. The reactor volume is 2m3 .
The inlet flow rate is 0.5 m3 /hr and the
inlet concentration conversion rounded to
2 decimal place is____________
(GATE 2017)
21.The elementary second -order liquid
phase reaction A + B → C + D is carried
It is observed that when the recycle ratio out in an isothermal plug flow reactor of
R=0.5, the exit conversion X Af = 50%
2m3 volume. The inlet volumetric flow
when the recycle ratio is increased to R=2, rate is 10m3 /h. The initial concentrations
the new exit conversion (in percent) will of both A and B are 2kmol / m3 . The rate
be:
constant is given as 2.5 m3 kmol −1h −1 . The
(a) 50.0 (b)54.3 percentage conversion of A is _________
(c) 58.7 (d)63.2 (GATE 2018)
(GATE 2015)
ANSWER KEYS
1 2 3 4 5 6 7 8 9 10
(A) (C) (A) (C) (C) (D) (C) (C) (D) (D)
11 12 13 14 15 16 17 18 19 20
(C) (C) (D) (B) (A) (C) (A) (A) (0.267) (0.80)
21
(50%)
EXPLANATIONS

1.Solution:- (a) 1 C A0V


τ= =
s FA0
=
rB k1C A − k2CB
FA0 C A0 (−rA )
The concentration and rate of reactant B is ∴ = =
related as per Eq.(1) the concentration of V τ XA
B will increase as A react to form B and C A0 KC A0 (1 − X A )
then concentration of B will decrease as B =
τ XA
reacts to form C. Thus, it shows a
maximum point in between. But X
∴τ k = A
concentration of B will not show a 1− X A
maximum, if rate of disappearance of B is
4. Solution: (c)
higher than formation, i.e, when
For case (1),
K 2CB 0 > K1C A0
Given , C Ao = 2kmol / m3
2.Solution:- (c)
Vo = 1 m3 / h
dC
−rA =− A =k1C A , C A = 0.5 kmol / m3
dt

For batch reactor, first order reaction V XA


For MFR, =
FAo (−rA )
dC Ao (1 − X A )
− =
k1C Ao (1 − X A )
dt V X
Or = A ...(1)
dX A VoC Ao kC A
= k1 (1 − X A )
dt
First order kinetics.
3. Solution: (a)
CA
Here, X A = 1 − 0.75
For CSTR, C Ao

Input = output+ disappearance + ∴


V
=
0.75
Accumulation 1× 2 k × 0.5

F= FA0 (1 − X A ) + (−rA )V Or kV = 3....(2)


A0

FA0 X A = (−rA )V =
For case (2), C A' o 4=
m3 / m3 , Vo' 3m3 / h
C A' o =0.5 kmol / m3 V= kC A2V +
d (C AV )
0 C A0
Or dt
Here, X A' =0.875
C A0 = C AF
And from Eq.(1), Here, it is given that V0 =F and

V' 0.875 d (VC A )


= ∴ =FC AF − kC A2V
3 × 4 k × 0.5 dt
kV ' = 21...(3) 7.Solution:- (c)
From Eqn. (2) and (3), 1
A + B → C
V' 2
=7 Initial moles,  1        0.5           0
V
5. Solution:- (c)  1
1 − 1 + 
Here, ε A =  2 = −1
PR
 1 3
1 + 
Initial moles, C p0 0  2

After 40% Volume at point Q,

conversion, C p0 (1-0.4) 0.4 C p0 v A vo (1 + ε A X A )


=
 1 
V XP =
v1 vo 1 − X A 
For MFR, =  3 
FP (−rP )
But the temperature is changing. Thus we
V
=
XP need to find the volumetric flowrate at Q
VoCP 0.5CP − 0.125CR with temperature T1 = 360k .
τ 0.4
Or = With constant pressure
C po 0.5 × 0.6C po − 0.125 × 4C po
τ = 1.6 h V1 V2
= cos −1 θ
T1 T2
6.Solution:- (d)
1
V0 (1 − X A )
A→ B 3 V
= 2
300 360
(−rA ) =
kC A2 , 1
second order reaction =V2 1.2V0 (1 − X A )
3
Applying material balance for A,
Thus the concentration at point Q ,
Input = Output + Reaction + Accumulation
dN A
FA0 = 0 + (−rA )V +
dt
NA N A0 (1 − X A ) N A0 (1 − X A )
=
C = CA =
V2 1.2Vo (1 − 0.33 X A )
A
1.2V0 (1 − 0.33 X A )
1− X A  1− X A 
C A = 0.83C Ao ( ) C A = 0.83C A0 
1 − 0.33 X A 
 1 − 0.33 X A 
8.Solution: (c) 9. Solution:- (d)
A  2B A → 5B
Initial moles 1 0 Let initial number of moles of A in the
At equilibrium, 1 − X e 2 X e feed= nA

Given, =
−rA 0.02C A − 0.01CB Initial moles of inert in the feed = n1

For maximum limiting conversion of A, After complete conversion,


reaction should be in equilibrium. At
= n + nI
equilibrium, rate of forward reaction = Total number of moles B
= 5nA + nI
Rate of backward reaction
Or 0.02C Ae − 0.01CBe =
0 Given,
V final
=3
Vinitial
Or 0.02(1 − X e ) − 0.01(2 X e ) =
0
5nA + nI
1 ∴ =
3
Xe = nA + nI
2
nA = nI
With constant pressure,
Thus, minimum mole percent of the inert
v1 v2 nI
= =
in the feed ×100
= 50%
T1 T2 nA + nI
1
V0 (1 − X A ) 10. Solution: (d)
3 V2
∴ =
300 360
The rate of reaction
1 dC dCB
Or =V2 1.2V0 (1 − X A ) − A = = k1C A − k2CB
3 dt dt
dX A
Thus, the concentration at point Q, ∴ C A0 = k1C A0 (1 − X A ) − k2 (CB 0 + C A0 X A )
N dt
CA = A = k1C A0 − k2CB 0 − (k1 + k2 )C A0 X A ...(1)
V2
At equilibrium, X A = X Ae

dX Ae
=0
dt
∴ k1C A0 − k2CB 0 =(k1 + k2 )C A0 X Ae ...(2) Rate in = Rate out + Generation
+Accumulation
From Eqs.(1) and (2),
d
dX A ρ0V0 = 0 + 0 + ( ρ0V )
C A0 = (k1 + k2 )C A0 ( X A0 − X A ) dt
dt dV
dX A = V0
= (k + k )dt dt
( X Ae − X A ) 1 2 = V0t + C1
V
=t 0,=V 0
Integrating,
∴ C1 =0
XA t
dX A

0
=
( X Ae − X A ) ∫ (k
0
1 + k2 )dt Thus, V = V0t...(1)

Now, applying material balance for A,


[ − In( X Ae − X A )]0 =+
XA
(k1 k2 )t
Input =Output +Reaction +Accumulation
 X − XA 
ln  Ae =−(k1 + k2 )t
FA0 = 0 + (−rA )V +
dN
 X Ae  dt
X Ae − X A d (C AV )
= e= − ( k1 + k2 ) t
e − t /τ V=
0 C A0 kC AV +
X Ae dt

∴ Time constant for different substances dC A dV


Or V0C A0 = kC AV0t + + CA.
1 dt dt
as τ =
k1 + k2
dC A
Or V0C A0 = kC AV0t + V0 + C A .V0
11. Solution: (c) dt

Semi-batch reactor is shown in the figure. dC A  1 + kt  C A0


Or +  CA = ...(2)
dt  t  t
A→ B
Given, V0 = 1 L / min We need to solve this equation.
Integration factor
Initial volume V0 = 0
  1 + kt  
= exp  ∫   dt = exp(kt + Int )= te
kt

k = 1min −1   t  

−rA =
kC A (first order reaction) Solution of Eq. (2) can be written as

Since, the reactor is being filled C A0


continuously, the volume changes with
=
C Ate kt ∫ t
te kt dt + C2

time and at any time t, V (t) can be found


C A0
from overall mass balance of all species. =
C Ate
kt
e kt + C2 ...(3)
K
At,=t 0,=
CA 0 Or F=
A,in FB ,in + Fl ,in

C A0 Thus, according to option (c), it satisfies


C2 = −
K this result

C A0 ∴ FA,in : FB ,in : Fl ,in =


3 : 2 :1
From Eq. (3), =
C Ate kt (e kt − 1)
K
13. Solution:- (d)
CA 1 A→ B→C
= (1 − e − kt )
C A0 kt
A→ R
1
Or 1 − X A = (1 − e − kt ) Given, rR = k ' C A and=
rB kC A − kCB
kt
∴ (−rA )= kC A + k ' C A = (k + k ')C A
Putting values, k= 1 min −1 , t=2 min
1− X A =
0.43 Applying material balance for B.
X A = 0.57 FCB0 + (rB )V =
FCB

12. Solution:- (c) V


(kC A − kCB )τ =
CB (as =τ)
A + 3B → 2C + Inert F
Initial (F )
A,in (F )
B ,in 0 (F )I ,in
CB =
kC Aτ
1 + kτ
 F ×  FB ,in − 
 0.6 F  ( )
After  A,in    0.4FA,in kC Aτ
CB =
 0.8   A,in  (1 + kτ ) 1 + ( k + k ')τ 
Initial concentration of A, For maximum exit concentration of B,
FA,in dCB
FA,in + FB ,in + Fl ,in =0

Concentration of A after 20% conversion kC A0 (1 + kτ ) [1 + (k + k ')τ ] −
0.8 FA,in kτ C A0 (1 + kτ )( k + k ') + k 1 + ( k + k ')τ   =
0
0.6 FA,in + FB ,in + Fl ,in
1 + k ( k + k ' )τ 2 + ( 2k + k 1 )τ =
Given, the concentration of A remains
same throughout the reactor. ( k + k ' )τ + k ( k + k ' )τ 2
+ kτ + k ( k + k ')τ 2
FA,in 8.0 FA,in or k (k + k ')τ 2 =1
∴ =
FA,in + FB ,in + Fl ,in 0.6 FA,in + FB ,in + Fl ,in τ=
1
k (k + k ')
=
0.2 FA,in 0.2 FB ,in + 0.2 Fl ,in
14. Solution:- (b) 15. Solution:- (a)
A → product For autocatalytic reaction, A + R → R + R

−rA =
kC1/2
A
Initially, the reaction rate will be rapid due
to production of R then with fall in
For isothermal batch reactor concentration of a, the rate will be
dC A declining.
− =
(−rA )
dt Thus, the curve will look like as given
dC below.
− A = k C A ...(1)
dt
16.Solution: (c)
CA
We have, X A = 1 − For batch reactor zero order reaction
C A0
dc A
=K
∴ C A0 dX A =
−dC A dt
cA t
or ∫ dc A = − ∫ dt
From Eq. (1), c 0

dX A
C=
A0 k C A0 (1 − X A ) c A − c A0 =
−kt
dt
dX A = 2 − 0.5 × 3 = 0.5mol / m3
C A0 . = kdt...(2)
1− X A 17.Solution:(a)
0.75 10 FA0 X A
C A0 . ∫
dX A ForCSTR.volumeV =
1 − X A ∫0
= kdt −rA
Or 0 From
dx
0.75
ForPFRvolumeV = FA0 ∫ A
C A0 .  −2 1 − X A  =
10 −rA
0

plot it is clear that volume of ideal CSTR


C A0 [ −1 + 2] =10k (area) is higher than of the ideal PFR. For
positive order reactions PFR is always
C A0 preferred over CSTR
Or = 10...(3)
k
18.Solution:- (a)
Now, again from Eq.(2)
Mole balance
1 t
dX A Input-Output-disappearance =
C A0 .∫
1 − X A ∫0
= kdt
0 Accumulation

C A0 1
At steady state,
= −2 1 − X A  = (QC A0 + RQC A ) − ( RQ + Q)C A =
k  0 t kC AV
=
t 10 × 2
t = 20 min
⇒ QC A0 − QC A =
kC AV
kC Aτ
⇒ C A0 − C A = 20. Solution: 0.80
C A0 − C A
τ= Design equation for CSTR is
kC A0 (1 − X A )
XA XA C A0 − C A V C A0 − C A
=τ ⇒
= τk τ
= = =
k (1 − X A ) 1− X A (−rA ) V0 0.005C A2
τk Given: V = 2m3 , v0 = 0.5m3 / hr
XA =
1+τ k
C A0 = 1000 mol/m3
So , conversion X A is independent of
Recycle ratio R C A0 X A
τ=
0.005 × C αA (1 − X A )
2
So final conversion =50%
XA
19. Solution =
0.005 × C A0 (1 − X A )
2

For reaction given, rate equation can be


V 2 XA
representing as, −r=
A k1 C A − k2 C B = =
v0 0.5 0.005 ×1000 × (1 − X A )2
C AO ⋅ X A C AO ⋅ X A
For MFR, =τm = (1 − X 2 ) 2 =
0.05 X A ,
(−rA ) k1 C A − k2CB
C A0 (1 X A ) and CB =
C A =− C AO . X A 1 − 2 X A + X A2 =
0.05 X A

XA X A2 − 2.05 X A + 1 =0
So , τ m =
k1 (1 − X A ) − k2 X A X A = 0.80
So, for case – I 21.Solution: 50%
0.2
=1 = ⇒ 4k1 − k2 1 (I) XA
k1 (1 − 0.2) − k2 (0.2) V dX A
For case – I FA0
= ∫ kC C
0 A B

=τm 1sec ⇒=CA 8 l mol


xA
2 dx A
8
X A =1 − =0.2 10 × 2
= ∫ 2.5C A 0 (1 − x A )(C A 0 − C A 0 x A )
10 0

For case – II xA
1 dx A
=τm 5sec ⇒=C A 5 mol l
10
= ∫ 2.5 ( 2 ) (1 − x )2(1 − x A )
5
0 A

X A =−
1 =0.5 1 1
xA
dx A
10
For case – II
=
10 10 ∫ (1 − x
0 A )2
0.5
=5 = ⇒ k1 − k2 0.2 (II)
k1 (1 − 0.5) − k2 (0.5)
On solving equation (I) and (II)
k1 = 0.2667 sec −1
xA
1 
1= 
1 − X A 0
1
=1 −1
1− X A
1 1
= 2 ⇒ 1− X A =
1− X A 2
1
X A= = 0.5= 50%
2
3 DESIGN OF MULTIPLE REACTOR SYSTEM

In this chapter we will deal with systems The rates are always calculated at the exit
where there is more than one type of concentration of the respective reactors.
reactors. Reactors can be connected in We can also write the above relations in
series or parallel and the reactors can be of terms of conversion. We will define
same type or different type. For simplicity conversion with respect to the inlet to the
we will stick to constant volume reaction first reactor, therefore all the conversions
systems only. So we have the following are with respect to CAo.
combinations
C Ao X A1
C Ao (1 − X A1 ) & τ 1 =
C A1 =
• MFRs in series −rA1
• PFRs in series C Ao ( X A1 − X A 2 )
• MFR & PFR in series C Ao (1 − X A 2 ) & τ 2 =
C A2 =
−rA 2
• MFR in parallel
• PFR in parallel 3.1.1 FIRST ORDER REACTION (CVRS)
IN A SERIES OF MFRs
The rate law is independent of type and
size of reactor, so order and rate constant Assuming that the reactors are at different
will not change from one reactor to other. temperatures T1,T2 ……and of different
volume V1,V2……… Hence for a first order
3.1 MFRs in series
reaction
C Ao − C A1 C Ao
=τ1 = ; C A1
k1C A1 1 + k1τ 1
C A1 − C A 2 C A1 C Ao
=τ2 = ; C A2 =
k2C A 2 1 + k2τ 2 (1 + k1τ 1 )(1 + k2τ 2 )
C Ao
∴ C AN =
(1 + k1τ 1 )(1 + k2τ 2 ).........(1 + k Nτ N )
For the first MFR
1
X AN = 1 −
C Ao − C A1 (1 + k1τ 1 )(1 + k2τ 2 ).........(1 + k Nτ N )
τ1 =
(−rA )1

For the second MFR If temperatures of all the tanks and volume
of all the tanks are same, then the
C A1 − C A 2
τ2 = expression simplifies to
(−rA ) 2
C Ao
C AN =
(1 + kτ i ) N
1
X AN = 1 −
(1 + kτ i ) N

Where τ i is the space time of individual


tank.
3.1.2 SELECTING BEST ARRANGEMENT
OF MFRs IN SERIES
Suppose two MFRs of unequal volumes are
connected in series. The conversion to
obtained from the combination is fixed i.e.,
X2 is fixed. The best arrangement is that
The
which minimizes the total reactor volume
shaded area will be minimum if the area
for the given conversion.
KLMN is maximum. Since point X1 is not
The question is what should be value of fixed, the area of KLMN is not fixed and we
intermediate conversion X1 to minimize want to maximize this area. The area of
the total volume. KLMN will be maximum if the diagonal of
the rectangle is parallel to the tangent to
the curve at X1. So it suggests that best
combination of reactors depends upon the
kinetics and hence the order of the
reaction. We can make following
conclusions
 For first order reaction, the two
MFRs should be of equal volume.
 For n>1 smaller MFR should be
followed by bigger MFR. So first
MFR should be smaller while the
second should be bigger
 For n<1, bigger MFR should be
followed by smaller MFR.
3.1.3 PFRs IN SERIES
When PFRs are connected in series, then
If we place the bigger MFR before the the combination of reactor behaves as a
smaller MFR, then the total volume is single PFR of Total volume V obtained by
given by the shaded area. adding the individual volumes of the PFRs.
 When order is less than one, n<1
then early mixing is preferred. We
will prefer that arrangement of
reactors where extent of mixing
The above combination can be treated as a decreases from first reactor to last
single PFR of volume V1+V2 and space time reactor. Hence
τ1 + τ 2 . large MFR → small MFR → PFR
3.1.4 DIFFERENT TYPES OF REACTORS 3.2 REACTORS IN PARALLEL
IN SERIES
When reactors are connected in parallel,
There are cases when PFR & CSTR are then they are connected such that the
connected in series. The question arises residence time in both reactants remains
that which combination is better. The same. By this way we can get the most
combination which provides maximum economical combination.
conversion for a given total volume or the
When connected in parallel, the net
combination which has a minimum total
volume required for a given conversion volumetric flowrate and net molar
becomes the preferred choice of rectors. flowrate gets divided into different parts
(equal to number of reactors), not
The results can be summarized as
necessarily in equal parts. However the
Before understanding the best concentration of any part remains the
arrangement, let us understand the extent same. The main stream is divided into
of mixing. We know that in an ideal PFR different parts such that each divided
the extent of mixing is zero. So whatever is branch has same residence time inside the
the size of PFR, smaller or bigger, the reactor. Let us consider the following
extent of mixing remains zero. But in case example of two MFRs connected in parallel
of MFRs, the bigger MFR has greater extent
of mixing and as the size of MFR increases
the extent of mixing increases.
 For reaction order greater than 1
i.e., n>1 late mixing is preferred.
The combination in which extent of
mixing increases from first reactor
to the last reactor is preferred
combination. Hence
PFR → small MFR → L arg e MFR
Should be the choice.
 When the order of reaction is equal
to one, the arrangement of reactors The flowrates will be divided such that
has no effect on conversion. Hence
we will obtain same conversions in
any arrangement.
V1 V reaction proceeds, the conversion
= 2
FAo1 FAo 2 increases while concentration of reactant
τ1 τ2 V V2 decreases. Hence when plotted with
= = or 1 concentration, the graph should be
C Ao C Ao vo1 vo 2
observed from right to left.
o
Similarly, when two PFRs are connected in
series the flow rates will be divided so that
space time in both the branches remains
same.
3.3 BEST REACTOR ARRANGEMENT
1
For a given vs X A curve, the best
−rA
reactor arrangement can be found by
minimizing the total reactor volume. The
change in the type of reactor will take
place at that point of conversion where the
curve changes its nature.
In the following figure, it is clear that from
0 to X1 CSTR requires less volume, while
from X1 to final conversion PFR requires
less volume. So the best combination will
be a CSTR followed by a PFR.

1
The plot can also be between vs C A .
−rA

The same technique can be used to select


the best possible arrangement. But as the
GATE QUESTIONS

1. The conversion for a first-order liquid- COMMON DATA QUESTIONS


phase reaction A → B in a CSTR is 50%.
The liquid phase reaction A → products
If another CSTR of the same volume is
is to be carried out at constant
connected in series, then the percentage
temperature in a CSTR+ PFR) is 95%.
conversion at the exit of the second
The CSTR has a volume of 75 L. Pure A is
reactor will be
fed to the CSTR, at a concentration C A0
(a) 60 (b) 75 =2mol/L and a volumetric flow rate of 4
(c) 90 (d) 100 L/min. The kinetics of the reaction given
by
(GATE 2001)
mol
2. A liquid phase reaction is to be carried −rA =
0.1C A2 .
L − min
out under isothermal conditions. The
reaction as a function of conversion has 3. The conversion achieved by the CSTR
been determined experimentally and is is
shown in figure below. What choice of (a) 40% (b) 50%
reactor of combination of reactors will
require the minimum overall reactor (c) 60% (d) 80%
volume, if a conversion of 0.9 is desired?
(GATE 2008)
4. The volume of the PFR required (in
litre) is
(a) 380 (b) 350
(c) 75 (d) 35
(GATE 2008)
COMMON DATA QUESTION
The liquid-phase reaction A → B + C is
(a) CSTR followed by a PFR conducted isothermally at 50 oC in a
(b) PFR followed by a CSTR Continuous stirred tank reactor (CSTR).
The inlet concentration of A is 8.0 g-
(c) CSTR followed by a PFR followed by mol/L. At a space time of 5 min, the
CSTR concentration of A at the exit of CSTR is
(d) PFR followed by a CSTR followed by 4.0 g-mol/L. The kinetics of the reaction
a PFR g − mol
is −rA =kC A0.5 . . A plug flow
L − min
(GATE 2003) reactor of the same volume is added in
series after the existing CSTR.

© Copyright Reserved by Gateflix.in No part of this material should be copied or reproduced without permission
5. The rate constant (k) for this reaction (a) 2.64 (b) 2.00
at 50 oC is
(c) 1.64 (d) 0.54
 g − mol 
0.5

(a) 0.2  −1 (GATE 2010)


 min
 L 
COMMON DATA QUESTION
0.5
 L  −1 In an aqueous solution, reaction P → Q
(b) 0.2   min
 g − mol  occurs under isothermal conditions
following first order kinetics. The feed
 g − mol 
0.5

(c) 0.4  −1 rate is 500 cm3 / min and concentration


 min
 L  of P in the feed is 1.5 ×10−3 mol / cm3 . the
0.5 reaction is carried out in a 5 L CSTR. At
 L  −1 steady, 60% conversion is observed.
(d) 0.4   min
 g − mol 
9. The rate constant (in min −1 ) is
(GATE 2009) (a) 0.06 (b) 0.15
6. The concentration of A (in g-mol/L) at
(c) 0.21 (d) 0.28
the exit of the plug flow reactor is
(GATE 2011)
(a) 0.5 (b) 1.0
10. The 5L CSTR is replaced by five
(c) 2.0 (d) 2.5 CSTRs in series. If the capacity of each
(GATE 2009) new CSTR is 1 L, then the overall
conversion (in percentage) is
COMMON DATA QUESTION
(a) 65 (b) 67
A liquid phase reaction A → B is
conducted isothermally in a CSTR having (c) 73 (d) 81
a residence time of 2 s. The inlet (GATE 2011)
concentration of species A is 2 mol/L ,
and the outlet concentration is 1 mol/L. COMMON DATA QUESTION
The rate law for the reaction is
The first order liquid phase reaction
kC
−rA = A where, k=5mol/L-s. A → P is conducted isothermally in a
K + CA plug flow reactor of 5 L volume. The inlet
volumetric flow rate is 1 L/min and the
7. The value of K in mol/L is
inlet concentration of A is 2 mol/L
(a) 11 (b) 9
11. If the exit concentration of A is
(c) 5 (d) 2 0.5mol/L, then the rate constant (in
(GATE 2010) min −1 ) is

8. If the same reaction is conducted in a (a) 0.06 (b) 0.28


series of two CSTR with residence times (c) 0.42 (d) 0.64
1s and 0.2s, then the inlet concentration
of A in mol/L, required to attain an outlet (GATE 2012)
concentration of A in mol/L, is

© Copyright Reserved by Gateflix.in No part of this material should be copied or reproduced without permission
12. The plug flow reactor is replaced by Which amongst the following reactor
3 mixed flow reactors in series, of 2.0L configurations gives the lowest total
volume. The exact conversion of A (in %) volume of the reactor (S)?
is
(a) CSTR followed by PFR
(a) 35.9 (b) 52.5
(b)Two CSTR in series
(c) 73.7 (d) 94.8
(c) PFR followed by CSTR
(GATE 2012)
(d) A single PFR
13. A first order liquid phase reaction is
(GATE 2017)
carried out isothermally at a steady state
in a CSTR and 90% conversion is 15. A set of standard stainless steel pipes,
attained. With the same inlet conditions each of internal diameter 26.65 mm and
and for the same overall conversion, if 6000 mm length, is used to make a plug
the CSTR is replaced by two smaller and flow reactor by joining them in series to
identical isothermal CSTR in series, the% carry out degradation of polyethylene.
reduction in total volume, to the nearest Seven such pipes are required to obtain a
integer is conversion of 66% at 450 K. The
minimum number of standard 8000 mm
(a) 30% (b) 45%
long pipes of the same internal diameter
(c) 52% (d) 40% to be procured for obtaining at least 66%
conversion under the same reaction
(GATE 2013)
conditions is _________
14. The following reaction rate curve is
(GATE 2018)
shown for a reaction A → P . Here (−rA )
and X A represents reaction rate and 16. A CSTR and a PFR of equal volume
are connected in series as shown below
conversion, respectively. The feed is
to carry out a first-order, isothermal,
pure A and 90% conversion is desired.
liquid phase reaction A → P

The rate constant is 0.2 s −1. the space-


time is 5 s for both the reactors. The
overall fractional conversion of A is
_________ (rounded off to third decimal
place)
(GATE 2018)

© Copyright Reserved by Gateflix.in No part of this material should be copied or reproduced without permission
ANSWER KEYS
1 2 3 4 5 6 7 8 9 10
(B) (D) (C) (B) (C) (B) (B) (C) (B) (C)
11 12 13 14 15 16
(B) (C) (C) (A) (6) (0.816)
EXPLANATIONS

1.Answer:- (b) (1 − X A ) −1/2 =


2
1
For a CSTR, 1− X A =
4
τ XA 3
X A= = 0.75
=
CA −rA 4

τ XA Overall conversion =75%


=
C Ao KC Ao (1 − X A ) 2.Answer:- (d)

For first order reaction, 0 < X A < 0.4

XA Initially the rate of reaction is maximum


kτ =
1− X A and rate decreases with increase in
conversion or decrease in concentration of
Given, reactant i.e. reaction order is positive in
this region. For the positive order
X A = 0.5
reactions PFR is best.
0.5 0.4 < X A < 0.7
∴ Kτ =
1 − 0.5
Kτ = 1 After 0.4 conversion, the rate increases
with increase in conversion which shows a
Now, for two tanks of same volume in change in reactor type. This indicates
series, reaction order is negative in this region.
CSTR should be selected for this region.

(1 − X A ) −1/2 − 1
=
N X A > 0.4

Putting values, Then after 0.7 conversion, rate again


decreases with increase in conversion
2 which again shows a change in reactor
(1 − X A ) −1/2 − 1
=
2 type i.e. reaction order is positive in this
Kτ = 2 Kτ i = 2 ×1) region. For the positive order reaction PFR
is best.
Thus for minimum overall reactor volume,
PFR followed by a PFR is correct option.
3.Answer:(c) V
0.875
dX A
V0C A0
= ∫ 0.1C (1 − X A ) 2
2
=
Given, C A0 2=
mol / L, V0 4 L / min 0.6 A0
V=350 L
V × 0.1× C A0
0.95
 1 
V = 75 L and −rA =
0.1C A2 = 
V0 1 − X A  0.6
V XA
For CSTR, =
FA0 (−rA ) 5.Answer: (c)

V XA For a CSTR,
=
V0C A0 0.1C A2 V XA τ
= =
FA0 (−rA ) C A0
75 XA
= C A0 X A C A0 − C A
4 × 2 0.1C A (1 − X A ) 2
2
(−r= =
A)
τ τ
75
=
XA C A0 − C A
8 0.1× 4(1 − X A ) 2 or kC A0.5 =
τ
X A2 − 2.27 X A + 1 =0 8−4
∴k × 4 =
X A = 1.67or 0.6 5
 g − mol 
0.5
−1
Since, conversion can never be greater k = 0.4   min
 L 
than 1, therefore X A =0.6 or 60%.
6.Answer:(b)
4. Answer: (b)
C A2 = ?
Here,
∴ C A= C A0 (1 − X A ) For PFR

=2(1 − 0.6) =0.8 mol/L C A2


dC A
Given, overall conversion of A by
τ=
C A1
∫ (−rA )
CSTR+PFR system is 95%. c A2

Thus,=
C A C A0 (1 − 0.95)
−∫
τ= A
0.5
= CA − CA
dC
kC A
1
0.5k
( 1 2
)
C A1

=2(1 − 0.95) =0.1 mol/L For same volume of PFR and MFR,
V dX A τ PFR = τ MFR
Now, for PFR, =∫
F (−rA )
∴5 =
( 4 − C A2 )
0.5 × 0.4
C A=
2
1g − mol / L
7.Answer:(b) 5C A1 C A0 − C A1
Now, for 1st reactor =
For CSTR, 9 + C A1 τ1

V XA τ 5 ×1.1 C A0 − 1.1
= = =
FA0 (−rA ) C A0 9 + 1.1 1
C A0 = 1.64mol / L
=
Given, τ 2=
s, C A0 2mol / L
9.Answer:(b) Given υ0 = 500cm3 / min
C A = 1mol / L C A= 1.5 ×10−4 mol / cm3
0
CA 1 X AC A0
∴ X A = 1− = ⇒ (−rA ) = V= 5L = 5000cm3 , X A =0.6
C A0 2 τ
X AC A0 For a CSTR,
kC A
=
k + CA τ V X
= A
1 FA0 − rA
×2
5 ×1 2
=
K +1 2 Or
V
=
XA
,
5 1 υ0C A0 KC A0 (1 − X A )
= ⇒ K =9mol / L
K +1 2
First order kinetics,
8.Answer:(c)
υ0 X A
k=
For CSTR, we have V (1 − X A )
XA τ Putting values,
=
(−rA ) C A0
500 × 0.6
K=
C A0 X A C A0 − C A 5000 × (1 − 0.6)
Or (−r=
A) =
τ τ K = .15 min −1
Now, for 2nd reactor 10.Answer:(c)
=τ 2 0.2,
= C A1 ?, C A2 1mol / L
For each CSTR,
kC
Also (−rA ) = τ=
V 1000
=
k + CA 1
υ0 500
Where, k=5 and K=9 τ 1 = 2 min

5C A2 C A1 − C A2 5 ×1 C A1 − 1 Total residence time


∴= =
or
9 + C A2 τ2 9 +1 0.2 τ = Nτ 1 = 5 × 2
C A1 = 1.1 τ = 10 min
For N CSTR in series,
1 Putting values,
1 − X A5 =
(1 + kτ 1 )5 1− X A =
1
1 (1 + 0.28 × 2)3
1 − X A5 =
(1 + 0.15 × 2)5
= = 73.7%
X A 0.737
X A5 = 0.73
13.Answer:(c)
11.Answer:( b)
For (Mixed flow reactor) MFR,
A→ P C − C A1
kτ 1 = A0
For 1st order reaction in PFR, C A1

V dx C A0
=∫ = −1
FA0 −rA C A1
C A0
xA = 1 + kτ 1 ____( I )
V dx A

= C A1
FA0 0
kC A0 (1 − x A )
V 1 Similarly, for 2nd MFR
=
or .In(1 − x A )...( I ) C A1
FA0 kC A0 = 1 + kτ 2 ____( II )
C A2
Here, V=5 L, C A0 =2mol/L, vo =1L/min
C A0
By Eqs (I) and (II) (1 + Kτ 1 )(1 + Kτ 2 )
=
=
FA0 C=
A 0 .v0 2mol / min C A2
C A = 0.5mol / L
C A2
But =
1− X A =
1 − 0.9 =0.1
=C C A0 (1 − x A ) C A0
Since, A
= 2(1 − x A )
0.5
And τ 1 = τ 2 for identical reactors,
Putting values in Eq. (i),
10= (1 − Kτ 1 ) 2
5 1 1 + Kτ 1 =
− log e (0.25) 3.16
2 k ×2
Kτ 1 = 2.16
⇒ k ≈ 0.28 min −1
%
Reduction in reactor volume
12.Answer:(c)
V − (V1 + V2 )
= ×100
C AN 1 V
For N MFR in series, =
C A0 (1 + kτ i ) N V − 2V1 kτ − 2kτ 1
= ×100
= ×100
v kτ
1
Or 1 − x A = 9 − 2 × 2.16
(1 + kτ i ) N = =×100 52%
9

Here, τ =
2
= 2 min, N= 3 14.Answer:(a)
i
1
15.Answer: (6)

π D2
7 pipe volume = 7 × 6 ×
4
To have same volume,
The no. of pipes 8m required =
π D2
7×6×
4
= = 6
5.25
π D2 4

4
16.Answer :- 0.816
C A0 − C A1
T=
KC A1
C A0 − C A1
5=
0.2C A1
C=
A0 C A1 + C A1
1 1
=
C A0 C A0C A0 (1 −=
xA1 ) C A0
2 2
C A1 = 0.5
xA 2
T dX A
C A0
= ∫ 0.5C
x A1 A 0 (1 − x A )
xA 2
dX A
5= ∫ 0.2(1 − X
0.5 A )
− ln [1 − X A ]0.5A 2
1=
x

=
1 ln(0.5) − ln(1 − x A 2 )
1 − X A2 =
0.18
= = 81.6%
X A 2 0.816
4 MULTIPLE REACTIONS

Till now we were dealing with single A 


k1
→ R ; rR = k1C An1
reactions where only one rate law exists.
A 
k2
→ S ; rS = k2C An2
Those reactions where more than one rate
law exists are called multiple reactions. We define instantaneous selectivity of R
w.r.t S as
Series reactions
rR dCR k1 n1 − n2
A  k1
→ B  k2
→ =
S = = CA
rS dCS k2
−dC A
−=rA = k1C A
dt  This expression of instantaneous
dCB selectivity remains same for CSTR
=
rB = k1C A − k2CB
dt as well as PFR.
dCC  We can also define overall
=
rC = k 2C B selectivity as
dt
total moles of R formed
And parallel reactions S=
total moles of Sformed
A 
k1
→R 4.1.1 MAXIMIZING SELECTIVITY
A → S
k2

Since selectivity is the ratio of moles of


rR = k1C A desired product formed to undesired
rS = k2C A product formed, therefore we always want
(−rA )= k1C A + k2C A to maximize selectivity. The expression for
selectivity contains two terms
And various other combination of series
and parallel reactions are considered as k1
which is temperature dependent and
multiple reactions. We will only discuss k2
simple series and parallel reactions. The
kinetics of series and parallel reactions are C An1 − n2 which is concentration dependent
already discussed in chapter 1.
We will analyse selectivity by keeping one
4.1 SELECTIVITY factor (out of temperature and
concentration) constant and varying the
Let us consider a parallel reaction where R
other factor.
is the desired product and S is the
undesired product. 4.1.2 EFFECT OF CONCENTRATION ON
SELECTIVITY
The concentration dependent term in the the product distribution for
definition of selectivity is raised to power reactions of the same order.
n1-n2. Hence, we have following cases The reactions can also be of type

CASE I: n1-n2 is positive A + B 


k1
→ R ; rR = k1C An1 CBm1
For maximizing selectivity, the A + B 
k2
→ S ; rS = k1C An2 CBm2
concentration inside the reactor has to be
Here selectivity can be defined in the
as high as possible. We know that in a
similar way as
CSTR, there is a sudden decrease in
concentration and it remains at the lowest k1 n1 − n2 m1 − m2
S= C A CB
value throughout the reactor. While in PFR k2
or bath reactors there is a progressive
decrease in concentration and Let us consider a case where
concentrations are at higher values in n1 > n2 & m1 > m2 . Now to keep selectivity
these reactors. So for this case PFR or maximum, both CA and CB should be kept
batch reactors should be preferred. high.
CASE II: n1-n2 is negative When n1 > n2 & m1 < m2 , CA should be kept
Because concentration is raised to high and CB should be kept low. This can be
negative power, hence to maximize achieved by starting with A adding B
selectivity concentration within the slowly to it.
reactor should be kept as low as possible. Similarly, when n1 < n2 & m1 > m2 CA should
Therefore, CSTR should be preferred.
be kept low while CB should be kept high.
CASE III: n1=n2 This can be achieved by starting with B
and adding A slowly to it.
In this case choice of reactors has no effect
on selectivity of desired product So the product distribution in parallel
reactions can be controlled by using the
 We can conclude from above
proper contacting pattern for the reacting
discussion that if the order of
fluids. This proper contacting pattern can
desired reaction is greater than the
be achieved in a batch or continuous
order of undesired reaction, PFR or
reactor as shown in the figure.
batch reactor should be preferred
and vice-versa. For batch process,
 For reactions in parallel, the
concentration level of reactants is
the key to proper control of
product distribution. A high
reactant concentration favours the
reaction of higher order, a low
concentration favours the reaction
of lower order, while the
concentration level has no effect on
Case I: If Ea1 > Ea 2 , activation energy of
desired reaction is more than the
activation energy of undesired reaction.
For this case with increase in temperature
the rate constant of desired reaction will
increase more than that of undesired
reaction. Hence to increase the selectivity
temperature should be high.
Case II: If Ea1 < Ea 2 , activation energy of
desired reaction is less than the activation
These contacting patterns can also be
energy of undesired reaction. If
achieved in continuous reactors. The
temperature is increased, then the rate of
reactant whose concentration has to be
undesired reaction will increase to a
kept high should be added to a PFR and the
greater extent. Hence to maximize
reactant whose concentration is to be kept
selectivity temperature should be as low
low should be added slowly throughout
as possible.
the PFR.
4.2 YIELD OF A PARALLEL REACTION
CA high CB low
Yield is defined to give quantitative
treatment to product distribution in a
parallel reaction. It can be used to find how
many moles of desired and undesired
product has been formed. Considering the
simple parallel reaction

A 
k1
→ R ; rR = k1C An1
A 
k2
→ S ; rS = k2C An2
4.1.3 EFFECT OF TEMPERATURE ON
SELECTIVITY 4.2.1 FRACTIONAL YIELD

k1 Fractional yield is defined within the


The term also affects selectivity and reactor and its expression is same for both
k2
PFR and CSTR. Fractional yield is defined
depends upon conditions of temperature as
inside the reactor. We know that as
temperature increases, the rate constant rR dCR k1C An1
φR =
= =
increases but this increase takes place for A −rA dCR + dCS k1C An1 + k2C An2
both the reactions. So we have the
rS dCS k2C An2
following cases φS
= = =
A −rA dCR + dCS k1C An1 + k2C An2
Note that −rA = rR + rS is valid only for the N=
R N Ro + Φ R ( N Ao − N A )
|MFR
above reaction. For different A

stoichiometry proper relation has to be N=


S N So + Φ S ( N Ao − N A )
|MFR
A
derived.
For CVRS system
Overall yield can also be defined as C=
R CRo + Φ R (C Ao − C A )
|MFR
A
moles of R formed N R − N Ro
=ΦR = C= CSo + Φ S (C Ao − C A )
moles of A reacted N Ao − N A
S
|MFR
A A

moles of S formed N S − N So
=ΦS = 4.2.3 OVERALL YIELD FOR PFR
A moles of A reacted N Ao − N A
In a PFR there is a gradual decrease in
For constant volume reaction system concentration, due to which there exist
different values of concentration and
moles of R formed CR − CRo
=ΦR = hence different values of fractional yield
moles of A reacted C Ao − C A
A
inside PFR. Overall yield for a PFR can be
moles of S formed CS − CSo calculated by taking average of these
=ΦS =
A moles of A reacted C Ao − C A different fractional yields.

 Overall yield for reactors can be C Af

found by taking average of ∫φ R dC A


−1
C Af

fractional yield. Φ =φ Φ R |PFR =φR = = ∫φ


C Ao A
dC A
C Ao − C Af
C Af R

∫ dC
A A C Ao A

4.2.2 OVERALL YIELD FOR A MFR C Ao


A

In a MFR there is a sudden drop in


The amount of products formed in PFR can
concentration, and there is only one value
be calculated as
of concentration throughout the MFR.
Therefore only one value of fractional N=
R N Ro + Φ R ( N Ao − N A )
| PFR
yield exists inside the reactor. A

N= N So + Φ S ( N Ao − N A )
This is the value of fractional yield which
S
| PFR
A
is calculated at the exit concentration of For CVRS system
the MFR. Hence the average of the
C= CRo + Φ R (C Ao − C A )
fractional yield is equal to this value.
R
| PFR
A

Φ MFR = φ R = φ R at exit concentration C=


S CSo + Φ S (C Ao − C A )
| PFR
A
A A
C Af
k1C Afn1
= =
C R CRo − ∫φ R dC A
k1C Afn1 + k2C Afn2 C Ao A
C Af

Therefore, total moles of R formed in a =


C S CSo − ∫φ S dC A
MFR is given by C Ao A
 Note that Φ R = 1− Φ S . This ( rR )V = vCR
( k1C A − k2CR )V =
A A
vCR
relation is valid only for the given
stoichiometry. k1C Aτ m − k2C=
V
Rτ m CR Q= τ m
v
4.3 OPTIMUM τ FOR A SERIES
REACTION IN A MFR
(1 + k2τ m ) CR =
k1C Aτ m

The kinetic equations for series and k1C Aτ m


CR =
parallel reactions developed earlier are (1 + k2τ m )
only valid for batch reactor and constant k1C A0τ m
volume plug flow reactors. However, if CR =
(1 + k1τ m (1 + k2τ m )
series or parallel reaction takes place in a
MFR, the equation should be obtained by to maximize R,
writing material balance on the reactor. dCR
=0
For the MFR , material balance gives dτ m
we obtain,
In – out – disappearance by reaction = 0
1
Writing the equation for reactant A gives τ m max =
k1.k2
FAo − FA − (−rA )V =0
CR max = 1
vC Ao − vC A − k1C AV =
0 CAo 2
 k 1/2 
Considering
V
=τM  2  + 1
v  k1  
CA 1
=
C Ao 1 + k1τ M

Writing the balance for product R


−rA =
k1C A
=
rR k1C A − k2CR
We know that
C AO
CA =
1 + k1τ m

Material balance on R
Input + Generation of R = Output
GATE QUESTIONS

COMMON DATA QUESTION (c)Use batch reactor or plug flow reactor


when a>b
The following gas phase reactions are
carried out isothermally in a CSTR (d) Use CSTR with a high conversion
when a>b (GATE 2004)
A → 2 R=
; r1 k1 p A =
; k1 20mol / ( s − m3 − bar )
4. Consider the following elementary
A → 3S=
; r2 k2 p A=
; k2 40mol / ( s − m3 − bar ) reaction network

Total pressure =1 bar, FA0 = 1 mol / s;


feed is pure A
1. What is the maximum possible value of
FR (mol/s)?

(a) 1/3 (b) 1/2


(c) 2/3 (d) 2
The activation energies for the individual
(GATE 2003) reactions are
2.The volume of a CSTR required for a E1 =100kJ/mol, E2 =150kJ/mol,
fractional conversion of A equal to 0.3
due to the first reaction is E3 =100kJ/mol, and E4 =200kJ/mol.

(a) 0.011 (b) 0.021 If the feed is pure A and the desired
product is C, then the desired
(c) 0.275 (d) 0.375
temperature profile in a plug flow
(GATE 2003) reactor in the direction of flow should be
3. Pick the wrong design guideline for a (a) Constant at low temperature
reactor in which the reaction A → R
(b) Constant at high temperature
(desired) and A → S (undesired) are to
take place. The ratio of the reaction rate (c) Increasing
r k  (d) Decreasing
is R =  1  C Aa −b
rS  k2 
(GATE 2006)
(a) Use high pressure and eliminate inert
COMMON DATA QUESTION
when a>b
The following liquid phase reaction is
(b) Avoid recycle when a>b
taking place in an isothermal batch
reactor.

A 
k1 (first order)
→ B 
k 2 (zero order)
→C
Feed concentration =1mol/L

© Copyright Reserved by Gateflix.in No part of this material should be copied or reproduced without permission
5. The time at which the concentration of favour the formation of the desired
B will reach its maximum value is given product indicated in the reaction scheme
by given below

1  k1  1 k  A 
1
→ R 
3
→ S ( Desired )
(a) t = In   (b) t = In  2 
k1  k2  k2 − k1  k1 
A 
2
→U
1  k2  1  k1 
(c) t = In   (d) t = In   n1 E1 n2 E2 n3 E3
k2  k1  k2  k2 
2 25 1 35 3 45
(GATE 2007)
(a) High C A0 , increasing t, plug flow
6. The time at which the concentration of reactor
B will become zero is given by the
following equation (b) Low C A0 increasing t, plug flow
reactor
(a) 1 − e − k1t =
k2t (b) t = infinite
(c) High C A0 decreasing t, mixed flow
1 1 reactor
(c) t = (d) t =
k2 k1
(d) High C A0 decreasing t, plug flow
(GATE 2007) reactor
7. The following liquid phase reaction is (GATE 2007)
taking place in an isothermal CSTR
9. Reactant R forms three products X, Y
A → B → C
k1 k2 and Z irreversibly, as shown below

2A 
k3
→D
Reaction mechanism is same as the
stoichiometry given above. Given,

k1 =1 min −1 ;k 2 =1 min −1 ;k 3 =0.5 L/mol-min;


CA0 =10 mol/L

CB0 =0 mol/L and CB =1 mol/L, the


solution for F/V (flow rate/reactor The reaction rates are given by
volume in min −1 ) yields. =
rX C= k y CR1.5 and rz = k z CR . the
R , rY

(a) 6.7 (b) 6 and 0.5 activation energies for formation of X. Y


and Z are 40, 40 and 5 kJ/mol
(c) 2 and 4/3 (d) 8
respectively. The pre-exponential factors
(GATE 2007) for all reactions are nearly same. The
desired conditions for maximizing the
8. Determine the level of C A0 (high , low,
yield of X are
intermediate), temperature profile (high,
low, increasing, decreasing), which will (a) high temperature, high concentration
of R

© Copyright Reserved by Gateflix.in No part of this material should be copied or reproduced without permission
(b) low temperature, low concentration 12. What is the % conversion of A, to the
of R nearest integer, so that the concentration
of S in the exit stream is 11.8 mol / m3 ?
(c) low temperature, high concentration
of R (a) 90% (b) 40%
(d) high temperature, low concentration (c) 60% (d) 50%
of R (GATE 2011)
(GATE 2013)
10. Consider the reaction scheme shown
below: A → B → C 13. In a laboratory batch setup, reaction
of P over a catalyst was studied at
Both the reactions are of first-order. The various temperatures. The reactions
activation energies for k1 and k2 are 80 occurring are P → 2Q; P → R
and 20 kJ/mol respectively. To maximize
At the end of one hour of operation, the
the yield of B, it is preferable to use
batch contains X P , X Q and X R mole
(a) CSTR and high temperature fraction of P, Q and R components,
(b) PFR and high temperature respectively. The mole fractions of
product components ( X Q and X R ) were
(c) CSTR and low temperature
found to vary linearly temperature as
(d) PFR and low temperature given in the figure.
(GATE 2012)
COMMON DATA QUESTION
Liquid reactant A decomposes as follows

A → R, rR =
k1C A2 k1 =0.5 m3 /mol-s

A → S, rs = k 2 CA k 2 =1 s −1

An aqueous feed of composition


= =
C A0 30 mol / m3 , CR 0 2mol / m3 . If the yield of Q based on reactant P
Cs 0 = 1mol / m3 consumed (YQ ) at 250 C was found to
enters a CSTR in which the above be0.40,then the value of YQ at 600 C is
reaction occur. Assume isothermal and __________(rounded off to second decimal
steady state conditions. place). (GATE 2018)
11. If the conversion of A is 80% the
concentration of R in the exit stream in
mol / m3 , to the nearest integer, is

(a) 40mol / m3 (b) 50mol / m3

(c) 30mol / m3 (d) 20mol / m3


(GATE 2013)

© Copyright Reserved by Gateflix.in No part of this material should be copied or reproduced without permission
ANSWER KEYS
1 2 3 4 5 6 7 8 9 10
(C) (B) (D) (D) (A) (A) (B) (A) (D) (B)
11 12 13
(D) (A) (0.4)
EXPLANATIONS

1.Answer:(c)
A → 2 R=
, r1 k1 p A =
, k1 20mol / s − m3 − bar V 0.3
=
A → 3=
S , r2 k2 p A=
, k2 40mol / s − m − bar 1 20 (1 − 0.3)
3

Total rate of reaction for A, V = 0.021 m3


( −rA ) = r1 + r2 = (k1 + k2 ) pA 3.Answer: (d)
r1 k1 20
∴ = = A → S (Undesired ) A → R( Desired )
( −rA ) k1 + k2 20 + 40
r1 1 rR  k1  a −b
= Given, =   CA
( −rA ) 3 rs  k2 

For 1 mol of A disappearing in A → 2 R ,2 a>b; PFR should be used CSTR is wrong to


mol of R produced. use when a>b. If R desired product

∴ Maximum possible value of 4.Answer:(d)


1 2
FR = × 2 = Given,
3 3
= =
E1 100 kJ / mol , E2 100kJ / mol ,
2.Answer:(b)
= =
E3 100 kJ / mol , E4 100kJ / mol ,
V XA
For CSTR, = When the feed is pure A, higher
FAo (−rA )
temperature of the reactor will favor
Here, X A =0.3, FAo =1 mol/s reaction 2 because E2 > E1. after
substantial amount of C is formed, we
But this conversion is only due to first don’t want reaction 4 to take place. Since
reaction. According to the previous E4 is largest, lower temperature will be
relation twice of this will react through the
desired so as to lower the formation D
second reaction Hence the conversion of A
from reaction 4.
is 0.9
Thus, decreasing temperature profile is
V XA
= correct answer.
FAo ( −rA )
5.Answer: (a)
V XA  p Ao = 1, 
=   A 
k1
→ B 
k2
→C
FAo k1 p Ao (1 − X A )  Pure Feed 
Given,
C A0 = 1mol / L At t=0, CB =0 (as there is no B in the feed)

For 1st order reaction, ∴ 0 =−1 − 0 + C


C =1
dC A
= −k1C A From Eq. (3) ,
dt
CA t 1 k2t − e − k1t
CB =−
dC A
∫ CA
= −k1 ∫ dt
Or C A0 0 Now, for C=
B 0,=
t ?

ln(C A / C A0 ) = −k1t ∴ 0 = 1 − k2t − e − k1t or 1 − e − k1t =


k2t
− k1t
=C A C=
A0 e (as C A0 1)
7.Answer: (b)
C A = e − k1t ...(1) Given,

Now, for B, A 
k1
→ B 
k2
→C
2 A 
k3
→D
dCB
= k1C A − k2 −rA= k1C A + 2k3C A2
dt
= k1e − k1t − k2 ...(2) −rB= k1C A + k2CB

rC = k2CB
For maximum value of concentration of B,
rD = k3C A
dCB
=0
dt Also given, k1 = 1min −1 , k2 = 1min −1 ,
L
k1e − k1t − k2 =
0 k3 = 0.5
mol − min
k2 =
C A0 10mol / L,=
CB0 0,=
e − k1t = CB 1mol / L
k1
F
=?
− k1t
ln e = ln(k2 / k2 ) V
1  k1  Applying material balance for A for CSTR,
t= ln  
k1  k2 
FC=
A0 FC A + (−(rA ))V
6.Answer:- (a) (Here, F is volumetric flow rate in L/min)
Form Eq. (2), =
Or F ×10 FC A (k1C A + k2C A2 )V
dCB
= k1e − k1t − k2 F k1C A + k2C A2
dt =
V 10 − C A
∫ dC = ∫ (k e
− k1t
B 1 − k2 )dt + C
−e − k1t − k2t + C...(3)
CB =
C A + 0.5C A2 At high temperature, k X > kZ
= ....(1)
10 − C A
At high concentration, rY > rx
Applying material balance B,
Thus, for maximizing the yield of X, the
FC=
B0 FCB + (−rB )V desired conditions are high temperature
0 = F ×1 + (1×1 − 1× C A )V and low concentration of R
F 10.Answer:(b)
= C A − 1....(2)
V
A 
k1
→ B 
k2
→C
From Eqs. (1) and (2),
Since, k = Ae − E / RT
C A + 0.5C A2 =
(10 − C A )(C A − 1)
The activation energy for k1 (80kJ/mol) is
1.5C A2 − 10C A + 10 =
0
higher than for k2 (20kJ/mol). Thus,
C A = 5.44 and 1.23
higher temperature is preferable to
F maximize the yield of B. Also, Plug Flow
= 4.44 and 0.23
V Reactor (PFR) should be used for better
The closest answer is 6 and 0.5. conversion rate.

8.Answer: (a) 11.Answer:(d)


Given,
A 
1
→ R 
3
→ S (desired )
A 
2
→U C A0 = 30mol / m3

As can be seen from the data, the CR 0 = 2mol / m3


activation energy is highest for desired X A = 0.8
product reaction. Thus, increasing ∴ C A= C A0 (1 − X A )
temperature will favour this desired = 30(1 − 0.8) = 6mol / m3
reaction. And for the same volume of PFR
and MFR,PFR is chosen for higher Applying material balance for A,
conversion rate. At last, higher initial
vC A= vC A + (−rA )V
concentration of A will give higher rate of 0

reaction resulting in more desired product vC A0 =vC A + (k1C A2 + k2C A )v


information. v(C A0 − C A ) = (k1C A2 + k2CS )v
Thus, high C A0 , increasing T, plug flow v(30 − 6)
= (0.5 × 6 × 6 + 1× 6)v
reactor is correct answer. V
= τ= 1s
v
9.Answer: (d)
Applying material balance for R,
− E / RT
k = Ae
k=
X k=
Y Ae −40/ RT and k = Ae −5/ RT
Z
vC=
R0 vCR − rR .v
CR=
0 CR − k1C A2τ
=
C R CR 0 + k1C A2τ
CR = 2 + 0.5 × 6 × 6 ×1
CR = 20mol / m3

12.Answer: (a)
Applying material balance for S,
=
vC so vCs (−rs )v
= Cs − k2C Aτ
Cso
=
1 11.8 − 1× C Aτ
C Aτ = 10.8

Now, applying material balance for A,


vC A=
0 vC A + (−rA )V

C A + (k1C A2 + k2C A )τ
Or C A0 =

C A (0.5C A + 1)C Aτ
Or C A0 =+

30 = C A + (0.5C A + 1) ×10.8 = C A = 3

% conversion of A,
CA 3
X A =−
1 =−
1 =0.9 =90%
C A0 30

13.Answer: 0.4(independent of temp)


5 NON-ISOTHERMAL REACTORS

5.1 THE ENERGY BALANCE EQUATION The above equation is written for a
continuous steady state reactor. Enthalpy
So far we were only dealing with
of products and reactants are calculated
isothermal reactors, temperature was not
with respect to a reference temperature.
changing. Hence there was no need to
This reference temperature should be
write energy balance for those reactors. In
same as the temperature at which
non-isothermal reactors, the temperature
enthalpy of reaction is known.
is not constant and hence along with
material balance equations, energy If energy balance equation has to be
balance equation is also required. written for a batch reactor, then energy
accumulation term should also be there.
 These non-isothermal reactors are
ideal reactors and therefore the (ENERGY OUT-ENERGY IN+ ∆H rxn N Ao X A )
design equations developed in the =(FINAL ENERGY OF SYSTEM-INITIAL
previous chapters are valid here. ENERGY OF SYSTEM)
 In batch reactors, the temperature
changes with time.  The energy balance equation can be
 In PFR the temperature changes written on the basis of mass or on
along the length of the reactor with the basis of moles. Before writing
increase in conversion. the energy balance we should
 In MFR, there is sudden change in check the units of specific heat. If
temperature just like sudden specific has the units of mass then
change in conversion and energy balance should be written
concentration. The entire MFR is at on mass basis and if specific heat
the same temperature. However, has the units of moles then energy
this temperature is different from balance should be written on the
the inlet temperature. basis of moles.
 Note that the energy balance 5.1.1 ENERGY BALANCE ON THE BASIS
equation for any reactor remains OF MASS
same.
The general equation of energy balance at
steady state is
∆H =
Qexternal
Consider the above reactor where q
∆H product − ∆H reac tan t + ∆H o rxn ( FAo X A ) = Qexternal denotes volumetric flow rate of the
streams, CP denotes the specific heat of the ( FACP + FB CP )(Tout − Tref ) − FAoCP (Tin − Tref )
streams which is assumed to be constant +∆H rxn FAo X A = Qext
with temperature. During a chemical
Substituting
reaction the total mass remains same.
Hence assuming density of the mixture to =
FA FAo (1 − X A )
remain constant we can write the FB = FAo X A
following energy balance equation at we get
steady state on the basis of mass CP (Tout − Tref )
XA =
∆H products − ∆H reac tan ts + ∆H rxn FAo X A = Qext −∆H rxn
ρ qC
 P (Tout − Tref ) − ρ qC
 P (Tin − Tref )
Note that the above relation is not
 Ao X A ) = Qext
+∆H rxn (qC universal. It is only valid for the given
reaction & adiabatic reactors and for the
Qext should be written according to the
above assumption. For any other reaction,
convention that heat added to the system
each component should be considered
is positive while heat withdrawn from the
separately with its individual specific heat.
system is negative. The enthalpy of
reaction should be calculated at the same 5.2 ADIABATIC REACTORS
reference temperature. The enthalpy of
For adiabatic reactors there is no heat
reaction should be used along with its sign
interaction between the system and the
depending on whether the reaction is
surroundings and hence Qext = 0 . By using
endothermic or exothermic.
energy balance we can conclude
5.1.2 ENERGY BALANCE ON THE BASIS
OF MOLES  If the reaction is endothermic and
takes place in adiabatic reactors,
When balance is written on the basis of then the exit temperature should
moles, we should be careful about each be lower than the inlet
and every component separately in the temperature.
reactant and product stream. Consider a  If the reaction is exothermic and
reaction A → B and assume that takes place in a diabetic reactor
C=PA C=PB CP . Assuming that reactant A then the exit temperature must be
is not converted completely we can write higher than the inlet temperature.
the following energy balance equation.  The above discussion also suggest
that an adiabatic reactor should
always be non-isothermal.
 We should note that we cannot
maintain temperature profile of
our choice in an adiabatic reactor.
The temperature profile is fixed in
an adiabatic reactor.
 If we want to maintain a desired
temperature profile, then heat
must be added or removed and it  For exothermic reactions, with
suggests that reactor cannot be increase in temperature, the
adiabatic. equilibrium constant decreases.
 The above results can be easily
proved by Vant-Hoff’s equation.
5.3 EQUILIBRIUM AND ACTUAL  The equilibrium conversion of a
CONVERSION OF A REACTION reversible reaction depends upon
temperature, pressure, presence or
When a reaction proceeds its conversion
absence of inert in the system.
always increases and reaches a certain
 With increase in temperature, the
value depending the time in a batch
equilibrium conversion increases
reactor or PFR and volume of MFR. This
for endothermic reactions while
conversion is called the actual conversion
decreases for exothermic reactions.
and depends upon the actual conditions
 With increase in pressure the
(time, volume, length of reactor etc).
equilibrium conversion shifts in
However, what is the maximum that direction where there is a
conversion that can be attained by a decrease in number of moles of
reaction is decided by the gaseous species and vice-versa.
thermodynamics and this value is called  Addition of inert to the system has
the equilibrium conversion. We can say the same effect as that of
that it is the maximum possible value of decreasing the pressure.
actual conversion provided sufficient  These results can be easily verified
residence time or volume of reactor. by using the Le-Chatelier’s
principle.
The maximum possible conversion of
irreversible reaction is 100%. But for In earlier chapters we were dealing with
reversible reactions it is less than hundred isothermal reactors and hence the
percent. Note that we always want higher equilibrium conversion was also constant.
values of equilibrium conversion because However, in non-isothermal reactions
if maximum possible conversion is high with increase in conversion the
then only we can achieve higher value of temperature changes due to which the
actual conversion. equilibrium conversion also changes.
Therefore, designing of non-isothermal
5.3.1 DEPENDENCY OF EQUILIBRIUM reactors are complex.
CONSTANT AND EQUILIBRIUM
CONVERSION
 Equilibrium constant of a reaction
depends only on temperature. It is
not affected by pressure or
presence or absence of inert.
 For endothermic reactions, with
increase in temperature,
equilibrium constant increases.
kf
Hence the ratio decreases or KC
kb
decreases.
5.4 REVERSIBLE REACTIONS IN
ADIABATIC REACTORS
If a reversible exothermic reaction takes
place in an adiabatic reactor, then we
know that with increase in conversion the
temperature of the reaction mixture will
increase. With increase in this
temperature the equilibrium conversion
decreases. There will be a temperature
For a reversible reaction we can obtain the when increasing actual conversion
expression for the equilibrium conversion becomes equal to decreasing equilibrium
by writing the expression of net rate and conversion. After this there will be no
equating it to zero. At equilibrium the net increase in actual conversion. This can be
rate which is forward rate minus the shown in the following figure
backward rate becomes zero and the
conversion at this condition becomes the
equilibrium conversion. This value of
equilibrium conversion is independent of
type of reactor.
 For a reversible reaction, if forward
reaction is endo thermic then
backward must be exothermic and
vice-versa.
 For a reversible reaction, the
endothermic path always has a
It can be understood from the above figure
higher activation energy than the
that if we want to increase the conversion
exothermic path. The path with
achieved then the inlet temperature must
higher activation energy is more
be reduced. Note that above figure is only
temperature sensitive.
for reversible exothermic reaction. The
 Hence for reversible exothermic
straight line in the above figure is obtained
reaction where backward reaction
from energy balance and has a slope of
is endothermic, with increase in
CP
temperature both the rate constant . For exothermic reactions this
increases. However the increase in −∆H rxn
backward rate constant is more. slope will be positive. For endothermic
reactions, the slope will be negative.
The conversion achieved can also be conversion decreases and
increased by doing inter-stage cooling. therefore we should use decreasing
temperature profile.
5.5 OPTIMUM TEMPERATURE
PROGRESSION
Now we will talk about those reactors
where we can maintain a desired
temperature profile. If we want to
maintain a temperature profile of our
choice then the reactor has to be non-
isothermal. Optimum temperature
progression means the temperature
profile that we want to maintain inside the
reactor to minimize the reactor volume.
This temperature progression can be a
constant temperature or a changing
temperature (with time in batch reactors
and with length in a PFR)
 For irreversible reactions whether
endothermic or exothermic, the
rate constant increases with
temperature and hence the rate
also increases. So for these
reactions the reactor should be
maintained at maximum possible
constant temperature. This
maximum temperature is decided
by the material of construction.
 For reversible reactions, there are
two factors. Rate and equilibrium
conversion.
 For reversible endothermic
reactions, with increase in
temperature the forward rate
increases and equilibrium
conversion also increases. Hence
these reactions should be carried
out at maximum possible constant
temperature.
 For reversible exothermic
reactions, with increase in
temperature equilibrium
GATE QUESTIONS

1. The reaction A → B is conducted in an 4. A CSTR is to be designed in which an


adiabatic plug flow Reactor (PFR). Pure A exothermic liquid phase first order
at a concentration of 2 k mol/ m is fed to
3 reaction of the type A → R is taking
the reactor at the rate of 0.01 m3 /s and at place. The reactor is to be provided with
a temperature of 500 K. If the exit a jacket in which coolant is flowing.
conversion is 20%, then the exit Following data is given
temperature (in Kelvin) is = C 5=
kmol / m3 ; X 0.5; feed
A0 A

∆H 2 =
−50000 KJ / Kmol , CPA temperature = reactor temperature =
= C=
PB 100 KJ / Kmol.K 400 C ; rate constant at
1min −1 ;(∆H ) =
400 C = −4kJ / mol ;
(a) 400 (b)500
ρ = 1000kg / m3 ;
(c) 600 (d) 1000
Cp = 4 J/g 0 C ; q = 10−3 m3 /min ( ρ and
(GATE 2001)
C p are same for the reactant and
2. A batch adiabatic reactor at an initial
product streams).
temperature 373 0 K is being used for
the reaction A → B . Assume the heat of The amount of heat to be removed is
reaction is -1 kJ/mol at 373 0 K and the (a)2/3 kW (b)1 kW
heat capacity of both A and B to be
constant and equal to 50 J/mol-K. The (c) 5/3 kW (d)4 Kw
temperature rise after a conversion of
(GATE 2003)
0.5 will be
5. An irreversible aqueous phase
(a) 5 0C (b) 10 0C reaction A + B → P is carried out in an
(c) 20 0C (d) 100 0C adiabatic mixed flow reactor. A feed
containing 4 kmol/ m3 of each A and B
(GATE 2002) enters the reactor at 8 m3 /h. If the
3. An exothermic reaction takes place in temperature of the exit stream is never
an adiabatic reactor. The product to exceed 390 K, what is the maximum
temperature_______________ the reactor feed inlet temperature allowed?
feed temperature. Data : Heat of reaction =-50kJ/mol,
(a) Is always equal to density of the reacting mixture =1000
kg/ m3 , specific heat of reacting mixture
(b) is always greater than
2 kJ/kg-K.
(c)is always less than
The above data can be assumed to be
(d) may be greater or less than independent of composition and
temperature.
(GATE 2002)

© Copyright Reserved by Gateflix.in No part of this material should be copied or reproduced without permission
(a) 190 K (b)290 K
(c) 390 K (d)490 K
(GATE 2004)
6. Consider a reversible exothermic
reaction in a plug flow reactor. The
maximum and minimum permissible
temperatures are Tmax and Tmin
respectively. Which of the following
temperature (T) profiles will require the (d)
shortest residence time to achieve the
desired conversion? (GATE 2004)
7.An endothermic aqueous phase first
order irreversible reaction is carried out
in an adiabatic plug flow reactor. The
rate of reaction
(a) is maximum at the inlet of the reactor
(b) goes through a maximum along the
length of the reactor
(a)
(c) goes through a minimum along the
length of the reactor
(d) is maximum at the exit of the reactor
(GATE 2004)
8. The gas phase reaction A → B + C is
carried out in an ideal PFR achieving
40% conversion of A. The feed has 70
mol% A and 30 mol% inerts. The inlet
(b) temperature is 300 K and the outlet
temperature is s400 K. The ratio of the
outlet to inlet molar concentration of A
(assuming ideal gas mixture and uniform
pressure) is
(a) 0.60 (b) 0.30
(c) 0.47 (d) 0.35
(GATE 2005)
9. A liquid phase exothermic first order
reaction is being conducted in a batch
(c) reactor under isothermal conditions by
removing heat generated in the reactor

© Copyright Reserved by Gateflix.in No part of this material should be copied or reproduced without permission
with the help of cooling water. The 11. The liquid-phase reaction A → B is
cooling water flows at a very high rate conducted in an adiabatic plug flow
through a coil immersed in the reactor reactor.
such that there is negligible rise in its
Data:
temperature from inlet to outlet of the
coil. If the rate constant is given as k, heat Inlet concentration of A =4.0 kmol / m3
of reaction (∆H ) , volume of the reactor
Density of reaction mixture
V, initial concentration as C A0 , overall
(independent of temperature) = 1200
heat transfer coefficient U, heat transfer
kg / m3
area of the coil is equal to A, the required,
cooling water inlet temperature Tci is Average heat capacity of feed stream
given by the following equation (independent of temperature)
=2000J/kg-K
(a) Tci= T −
( −H )VkC A0
Heat of reaction (independent of
UA
temperature) =-120kJ/mol of A reacting

(b) Tci= T−
( −H ) VkC A0 e − kt
If the maximum allowable temperature
UA in the reactor is 800 o K , then the feed

(c) Tci= T −
( −H )VkC A0e− kt temperature (in o K ) should not exceed ?
UAt (a) 400 (b) 500

(d) Tci= T −
( −H )VkC A0 (c) 600 (d) 700
UAt
(GATE 2009)
(GATE 2007)
12. For an exothermic reversible
10. The homogeneous reaction reaction, which one of the following
A + B → C is conducted in an adiabatic correctly describes the dependence of
CSTR at 800K so as to achieve a 30% the equilibrium constant (K) with
conversion of A. The relevant specific temperature (T) and pressure (P)?
heats and enthalpy change of reaction
(a) K is independent of T and P
are given by
C p , B = 100 J (mol − K ), C p ,C = 150 J / mol K (b) K is increases with an increase in T
and P
C p, A =
50 J (mol − K ), ∆H rxn =
−100kJ / mol
(c) K is increases with T and decreases
If the feed , a mixture of A and B, is with P
available at 550 o K , the mole fraction of (d) K decreases with an increase T and is
A in the feed that is consistent with the independent of P
above data is
(GATE 2012)
(a) 5/7 (b) 1/4
13. The elementary reversible
(c) ½ (d) 2/7 exothermic gas-phase reaction
A + 3B  2C is to be conducted in a non-
(GATE 2008)
isothermal, non-adiabatic plug flow

© Copyright Reserved by Gateflix.in No part of this material should be copied or reproduced without permission
reactor. The maximum allowable reactor (c) [ a ] is correct but [ r ] is false
temperature is Tmax . To minimize the
(d) Both [ a ] and [ r ] are false
total reactor volume, the variation of
temperature (T) with axial distance from (GATE 2014)
inlet(Z) will be
15. A liquid phase irreversible reaction
A → B is carried out in an adiabatic CSTR
operating under steady state conditions.
The reaction is elementary and follows
the first order law. For this reaction, the
(a) figure below shows the conversion ( X A )
of A as a function of temperature (T) for
different values of the rate of reaction
(−rA in molm −3 s −1 ) denoted by the
numbers to the left of each curve. This
figure can be used to determine the rate
(b) of the reaction at a particular
temperature, for a given conversion of A.

(c)

(d) (GATE 2012) The inlet stream does not contain B and
14. Determine the correctness of the the concentration of A in the inlet stream
following Assertion [ a ] and Reason [ r ] . is 5 mol/m3 . The molar feed rare of A is
100 mol/s. A steady state energy balance
Assertion: significant combustion of coke for this CSTR results in the following
takes only if it is heated at higher relation: =
T 350 + 25 X A where T is the
temperature in presence of air temperature (in K) of the exit stream and
X A is the conversion of A in the CSTR.
Reason: C + O2 → CO2 is an exothermic
For an exit conversion of 80% of A. The
reaction.
volume (in m3 , rounded off to the first
(a) Both [ a ] and [ r ] are true and [ r ] decimal place) of CSTR required is _____
is the correct reason for [ a ] (GATE 2016)
(b) Both [ a ] and [ r ] are true and [ r ]
is not the correct reason for [ a ]

© Copyright Reserved by Gateflix.in No part of this material should be copied or reproduced without permission
ANSWER KEYS
1 2 3 4 5 6 7 8 9 10
(C) (B) (B) (C) (B) (B) (A) (D) (B) (A)
11 12 13 14 15
(C) (A) (B) (B) (8)
EXPLANATIONS

1.Answer:(c) 4.Answer:(c)

=
Given C A0 2=
kmol / m3 , v0 0.01m3 / s =
Given C Ao 5=
kmol / m3 , X A 0.5

= =
T1 500 K, X A 0.2 = =
T 40 o
C , k 1min −1
C=
PA C=
PB 100kJ / mol − K −40kj / mol , ρ =
(∆H ) = 1000kg / m3
∆H =
50000 kJ / kmol of reacted C= 4kj / kg − K , q= 10m −3 / m3 min= Vo
p

Applying energy balance, Amount of heat removed


(V0C A0CPBT2 ) − (V0C A0CPAT1 ) + = VoC Ao (1 − X A ) × ∆H
( ∆H × V0 × C A0 X A ) =0
CPBT2 − CPAT1 =
∆H . X A 10−3 m3 mol kj
= × 5 ×103 3 × 0.5 × 40
60 s m mol
Putting values, 5 kj 5
= = kW
100T2 − 100 × 500= 50000 × 0.2 3 s 3

T2 = 600 5.Answer:-(b)

2.Answer:(b) Given,=C Ao 4=
kmol / m3 400mol / m3

Given, Vo = 8m3 / h
A→ B
=
(−∆H ) 50
= kj / mol , ρ 1000kg / m3 ,
=C pA C pB 50 j / mol − K =C p 2kj / kg − K
−∆H = −1kj / mol = −1000 j / mol
Applying energy balance for maximum
X A = 0.5
heat generated or all A reacted,
((∆T )C p =
∆Hτ × X A )
8m3 / h ×1000kg / m3 × 2kj / kg − K × ∆Tmax
(∆T )50 = 1000 × 0.5
=
8m3 / h × 4000mol / m3 × 50kj / mol
∆T =
100 C
3. Answer:(b) Or ∆Tmax =
100

For exothermic reaction in adiabatic Or 390 − ∆Tinlet =100


reactor, product temperature is always
greater than feed temperature. Tinlet = 290 K
6.Answer:(b)
Given reversible exothermic reaction in a 170 − 100
εA =
PFR. 100
A B = 0.7
 1− X A 
Option (a) Maximum temperature is C A = C Ao  
being maintained throughout the reactor.  1+ ε A X A 
When the product concentration C A = 0.47C Ao @ 300k
increased, rate of backward reaction also
increases due to higher T. Thus, overall But the outlet gas is at 400k
rate of reaction decreases. It is not a 300
desirable case. C A@ 400=k CA ×
400
Option (b) Initial higher temperature = 0.47 C Ao × 3 / 4
gives higher rate of forward reaction and = 0.35C Ao
decrease in temperature with length of the
reactor will not increase the backward C A@ 400 k
= 0.35
rate of reaction. The residence time will be C Ao
shortest.
Ratio of outlet to inlet molar concentration
Option (c) and (d) Initial lower 0.33
of= A = 0.47
temperature will give lower rate of 0.7
forward reaction and with increase in
temperature and product concentration, 9.Answer:(b)
the backward rate of reaction increases For batch reactor , applying the material
resulting in decrease in overall reaction balance,
rate.
Input = Output +Reaction + Accumulation
Thus, (b) is correct answer.
dN
0 = 0 + (− rA )V +
7.Answer :(a) dt

For adiabatic PFR, an endothermic 1st dN A


=−(−rA )V
order aqueous phase reaction would Or dt
maximum rate of reaction at the inlet of
the reactor because the temperature is dC A
= −kC A
maximum at the inlet and temperature Or dt (first order reaction)
decreases along the reactor due to CA t
endothermic reaction. dC A
∫ CA
= −k ∫ dt
or
8.Answer:(d) A → B
C A0 0
+ C inert
C A = C A0 e − kt ...(1)
Initial moles 70 0 0 30
After 100% 0 70 70 30 Now, Heat generated = Heat removed

(−rA )V (−∆
=H ) UA(T − T0 )
kC AV (−∆
=H ) UA(T − T0 ) Thus, the mole-fraction of A in the feed
Or nA 5
= =
kV nA + nB 7
.C A0 e − kt (−∆H=
) (T − T0 )
Or UA 11.Answer:(c)
(−∆H )kVC A0 e − kt A→ B
Tci= T −
Or UA
Given C A0 = 4kmol / m3

ρ = 1200kg / m3
10.Answer:(a) =CP 2000 J / kg − K
A + B→ C
∆H x =
−120kJ / mol of A reacting
Initial moles nA nB 0
Input-Output + Generation =
Moles after 30% conversion nA − 0.3nA Accumulation
nB − 0.3nA 0.3nA ρ .V0 .CPT1 − ρV0CPT2 + V0C A .∆H x =
0
0

Applying the energy balance for the


adiabatic CSTR, 1200 × V0 × 2000(T1 − T2 ) + V0 × 4 ×120 ×106 =0
Input = Output +generation
kg m3 J
Units 3 × × ×K
∑ n=
C ∆T ∑ n
in p 1 out
C p ∆T0 + ∆H r×nε m s kg − K
nA − (nA − 0.3nA ) m3 kmol kj mol J
=ε = 0.3 =
nA no . Of moles × 3 × ×106 ×
1 s m mol kmol kJ
of A reacted
24 × 105 (T1 − T2 ) + 480 × 106 =
0
Thus, putting values
T1 − T2 =−200
T2 =
−200 + 800
T2 = 600

12.Answer:(b)
For change in temperature and pressure,
the equilibrium shift to minimize the effect
as per Le’s chatlier principal. But
equilibrium constant is independent of T
and P.

13.Answer:(c)
14.Answer:(b)
Both [a] and [r] are true but [r] is not
correct reason for [a].

15.Answer: (8)
Given conditions

C Ao = 5mol / m3
FAo = 100mol / sec

Temperature profile is given as


=
T 350 + 25 X A

X A = 0.8

On putting this value we will get T= 370 K

At 370 K the value of (−rA ) corresponding


=
to this value (−rA ) 10mol / m 2 − s

For MFR we know

V X XA 100 × 0.8
== A ;V = FAo =
FAo −rA (−rA ) 10
V = 8 m3
6 NON-IDEAL REACTORS

In this chapter we will learn about non- immediately, while others spent so
ideal reactors, that is, reactors that do not much time in the reactor that they
follow the models we have developed for were almost completely consumed.
ideal CSTRs, PFRs, and PBRs. We describe The time the bubble spends in the
how to characterize these non-ideal reactor is termed the bubble
reactors using the residence time residence time.
distribution function E(t), the mean
residence time t , the cumulative  When a reactor is packed with
distribution function F(t), and the variance catalyst, the reacting fluid usually
σ 2 . The functions E(t) and F(t) will be does not flow through the reactor
developed for ideal PFRs, CSTRs. We will uniformly. Rather, there may be
then use these ideal curves to help sections in the packed bed that
diagnose and troubleshoot bypassing and offer little resistance to flow, and as
dead volume in real reactors. We will learn a result a major portion of the fluid
how to use the residence time data and may channel through this pathway.
functions to make predictions of Hence the molecules following this
conversion and exit concentration. pathway do not spend as much
Because the residence time distribution is time in the reactor as those flowing
not unique for a given reaction system, we through the regions of high
must use new models if we want to predict resistance to flow. We see that
the conversion in a non-ideal reactor. there is a distribution of times that
molecules spend in the reactor in
6.1 RESIDENCE TIME DISTRIBUTION contact with the catalyst.
We will consider following examples to
understand the need of residence time  In many continuous-stirred tank
distribution. reactors, the inlet and outlet pipes
are close together. In one operation
 In a gas-liquid continuous-stirred it was desired to scale up pilot plant
tank reactor, the gaseous reactant results to a much larger system. It
is bubbled into the reactor while was realized that some short
the liquid reactant is fed through an circuiting occurred, so the tanks
inlet tube in the reactor's side. The were modelled as perfectly mixed
continuous liquid phase could be CSTRs with a bypass stream. In
regarded as perfectly mixed, and addition to short circuiting,
the reaction rate was proportional stagnant regions (dead zones) are
to the total bubble surface area. The often encountered. In these regions
surface area of a particular bubble there is little or no exchange of
depended on the time it had spent material with the well-mixed
in the reactor. Because of their regions and consequently, virtually
different sizes, some gas bubbles no reaction occurs in this region.
escaped from the reactor almost
6.2 RTD EXPERIMENTS
RTD curves can be plotted by doing the
tracer experiments. Tracer is an inert
substance which is added to the reaction
mixture and its concentration is noted in
the exit stream with respect to time. A
tracer must be non-reactive, should not
stick to the reactor walls, should easily mix
with the reaction mixture and should be
easily detectable. Note that tracer stream
are also called stimulus response
techniques. Based on methods of addition  The area of C-curve is constant. We
No ∞
of tracer we have to types of tracer
experiments. The tracer can be injected in
know that ∫ dN = ∫ν oC (t )dt where
0 0
the form of a pulse or in the form of a step vo is the volumetric flow rate of the
input. stream. Hence area of C-curve is
No
6.3 PULSE RTD EXPERIMENT
vo
In this experiment a fixed amount (moles) Another important curve for the pulse
of tracer is injected all at once in a very experiment is the E-curve. E-curve is also
small amount of time. So the input profile called Exit-age distribution curve or RTD
is given as curve.
dN voC (t )dt C (t )dt
= =
No No No
vo
dN C (t )dt C − curve
= ∞
=
No Area of C − curve
∫ C (t )dt
0

dN C (t )dt C − curve
E (t )=
dt = ∞
=
Suddenly the concentration of tracer No Area of C − curve
increases to some value in the reactor and ∫ C (t )dt
0
then decreases and after infinite or very C − curve
long time when the entire tracer comes out E − curve =
Area of C − curve
of the reactor the concentration of tracer
becomes zero inside the reactor.

The curve plotted between concentrations  It shows that E-curve is


of tracer at the exit of the reactor vs time is proportional to C-curve and hence
known as the C -CURVE. A general C-curve the shapes of E and C curves are
for pulse experiment is given by similar.
 E-curve can be obtained from C-
curve by dividing the C-curve by its
own area.
dN
 Since E-curve = , it means that a
No
point on the E-curve denotes the
fraction of tracer. Hence E(t)dt
denotes the fraction of tracer which
spends time between t and t+dt
inside the reactor.
 The units of E(t) is time-1
 We can conclude that a point on the
E-curve denotes the fraction of
tracer that spends exactly time t  We know that in infinite time all the
inside the reactor. tracer should come out. Hence the
 If E-curve is integrated from 0 to t1, integral of E-curve from zero to
then it shows the fraction of tracer infinity should be one. Therefore
which spends time less than t1 the area of E-curve is equal to one .
inside the reactor. This fraction of  Since different molecules are
tracer can also be called as the spending different times inside the
fraction of tracer younger than t1 reactor, the average residence time
Fraction of tracer younger than t1 of all the molecules is given by
t1 ∞
= ∫ E (t )dt . This fraction is shown in the ∫ tE (t )dt ∞
0
t = 0

or ∫ tE (t )dt .
shaded area
∫ E (t )dt
0
0

 A vertical line through t should


divide the area of E-curve into two
equal curves.

Another distribution curve is Cumulative


exit age distribution curve or F-curve.
The F-curve and the E-curve are as related
t
Fraction of tracer older than t1 or the as F (t ) = ∫ E (t )dt .
fraction of tracer that spends time more 0

than t1 inside the reactor is given by It means a point on F-curve gives the same

information which is given by the area
= ∫ E (t )dt under the E-curve. Hence a point on the F-
t1
curve denotes the fraction of tracer that
has spent time less than t inside the
reactor. We can write the following
relations
t ∞
=
F (t ) ∫ E (t )dt & 1 −=
0
F (t ) ∫ E (t )dt
t

F-curve has the following nature


tracer is injected all at once, the “in” term
in the balance becomes zero. However,
with time the concentration of tracer
changes (decreases) inside the reactor.
Therefore, accumulation term should be
there. The equation becomes

in − out = accumulation
dC (t )
0 − vC (t ) =V
dt
6.3.1 MOMENTS OF RTD
The above differential equation can be
Nth Moment of RTD about origin is defined
solved with condition that at t = 0,
as
∞ C −t
C(t)=Co=No/V, which yields C (t ) = o e τ
µn = ∫ t n E (t )dt and nth moment of RTD τ
0 This is the C-curve of ideal CSTR. The C-
about its mean is defined as curve when divided by its area gives the E-

1 −τt
M= ∫ (t − t ) E (t )dt . Based on this
n
n curve E (t ) = e . It shows that the C-
0 τ
definition, we can define the following: curve and the E-curve are exponentially
 moment of RTD about origin decreasing functions of time for an ideal
∞ ∞
CSTR.
=µ0 ∫=
0
t 0 E (t )dt ∫ E (t )dt
0
which is

equal to area of the E-curve. 6.3.3 PULSE RTD FOR IDEAL PFR
 First moment of RTD about origin

In an ideal PFR, there is no axial mixing.
=µ1 ∫=
0
tE (t )dt t which is equal to
The type of input tracer signal that enters
the mean residence time. the reactor remains as it is at the exit of the
 The second moment of RTD about reactor. The PFR act as a lag and the output
its mean is equal to the variance of signal is delayed by time which is equal to
the RTD and is denoted by σ 2 . the space time of the PFR. The pulse input
∞ function is also sometimes represented in
M= σ= ∫ (t − t ) the form of dirac delta function. Hence
2 2
2 E (t )dt
0 output is also a dirac delta function
∞ ∞ ∞ delayed by τ P .
 σ = ∫ t E (t )dt + ∫ t E (t )dt − ∫ 2ttE (t )dt
2 2 2
t ) δ (t − τ P ) . The dirac delta function
E (=
0 0 0
follows these properties
σ= µ2 − t
2 2

6.3.2 PULSE RTD FOR IDEAL CSTR

The RTD for pulse experiment in an ideal


CSTR can be obtained by writing the
material balance on the tracer. Since the
0 when x ≠ 0
δ ( x) =  The input tracer profile is given by
∞ when x = 0

∫ δ ( x ) dx = 1
−∞

∫ g ( x ) δ ( x − τ ) dx =
−∞
g (τ )

The E-curve for an ideal PFR is shown


below

The Cstep curve for a CSTR can be obtained


by writing the material balance on the
tracer.
dCstep (t )
vCo − vCstep (t ) =
V
dt
By applying condition that at t=0 , Cstep=0,
we arrive to the following equation
−t
The F-curve for an ideal PFR is also shown (t ) Co (1 − eτ M )
Cstep=
below

6.4 STEP TRACER EXPERIMENT


The above curve when divided by Co
In step tracer experiment, tracer is generates the F-curve which is same as the
injected continuously at a concentration curve we obtained for the pulse input.
Co. The concentration of tracer in the exit
stream increases and after some time it
becomes equal to the concentration of
tracer in the inlet stream after which the
experiment can be stopped.

The C-curve of step experiment will be


denoted by Cstep(t). In pulse experiment we
divide the C-curve by its area to get E-
curve. In step experiment we divide Cstep
curve by Co and we get the F-curve.
 In an ideal PFR the step signal will
be delayed by a time equal to the
space time of the PFR. Hence Cstep
curve for a PFR will be a delayed
step signal.
 In step experiment, the mean
residence time is given by

∫ tdC step

t = 0

∫ dC
0
step

 It is interesting to note that the area The equation of exponentially decreasing


− ( t −τ p )
with respect to y-axis is Coτ and if 1 τM
part is given by e .
the area w.r.t y-axis is divided by Co τM
we get τ which is also equal to Similar RTD is obtained when CSTR is
mean residence time of the reactor. followed by a PFR.
6.5 RTD OF REACTORS IN SERIES When MFRs are connected in series, the
system behaviour becomes closer to PFR
When two ideal PFRs are connected in and its RTD also approaches towards PFR.
series, it will act as a single PFR of volume
given by adding the individual volumes. 6.6 RTD OF REACTORS CONNECTED IN
Hence the space time of the combination is PARALLEL
given by adding the space time of When reactors are connected in parallel,
individual reactors. the RTD is obtained by superimposing the
RTD of each branch. When two ideal PFRs
of different space times are connected in
Please note while plotting the RTD curves parallel, there will be two pulses in the
for pulse input. output as shown. Similarly if a PFR and a
CSTR are connected, the RTD will be
obtained by superimposing the individual
RTDs. Note that the RTD drawn is only for
a pulse experiment.

When a PFR is followed by a CSTR, then for


a pulse input the RTD curve is given by
6.7 NON-IDEAL REACTOR MODELS

The tank in series model and dispersion


model are the single parameter models.
The tank in series model considers a non-
ideal reactors as a combination of N tanks
(CSTR) in series. The value of N is to be
determined. The variance of RTD of ideal
PFR is zero while for ideal CSTR is τ 2 .

The Dispersion models explains the non-


ideality in a tubular reactors. There is no
axial dispersion in ideal PFR. Hence
dispersion number is zero. While there is
infinite mixing in CSTR and hence the
dispersion number is infinity. The peclet
number can be defined as inverse of
dispersion number.
GATE QUESTIONS

1. The mean conversion in the exit


4.For an ideal plug flow reactor the value
stream, for a second order, liquid phase
of the peclet number is
reaction in a non-ideal flow reactor is
given by (a) Zero (b) infinity

k2C A 0t (c) 1 (d) 10
(a) ∫ 1+ k C t
E (t )dt
0 2 A0 (GATE 2002)

1 5. A step input tracer test is used to
(b) ∫0 1 + k2C A0t E (t )dt explore the flow pattern of fluid through
a vessel of total volume equal to 1 m3

1 /min.
(c) ∫ 1+ k C 1 − E ( t )  dt
t
0 2 A0
Identify for each curve in Group I a
∞ suitable flow model from the list given
exp(−k2C A0t )
(d) ∫
0
1 + k 2 C A 0 t
E (t )dt under Group II.

(GATE 2001) Group 1 Group 2


2.The E-curve for a non-ideal reactor
P. Curve 1 1. PFR and CSTR in
defines the fraction of fluid having age
series
between t and t +dt Q. Curve 2
2. CSTR with dead space
(a) at the inlet
3. PFR in series with a
(b) at outlet
CSTR and dead space
(c) in the reactor
4. CSTR
(d) averaged over the inlet and outlet
(GATE 2001)
3.A pulse tracer is introduced in an ideal
CSTR (with a mean residence time τ ) at
time t=0. The time taken for the exit
concentration of the tracer to reach half
of its initial value will be
(a) 2 τ (b) 0.5 τ
(c) τ /0.693 (d) 0.69 τ
(GATE 2002)

© Copyright Reserved by Gateflix.in No part of this material should be copied or reproduced without permission
COMMON DATA QUESTION
The residence time distribution E(t) (as
shown below) of a reactor is zero until 3
min and then increases linearly to a
maximum value Emax 8min after which it
decreases linearly back to zero-at 15
min.

(a) P-4, Q-3 (b) P-4, Q-1


(c) P-2, Q-3 (d) P-2, Q-1
8. What is the value of Emax ?
(GATE 2003)
(a) 1/6 (b) 1/8
(c) ¼ (d) 1/3
6.For a packed bed reactor, the presence
of a long tail in the residence time (GATE 2005)
distribution curve is an indication of
9. What is the value of the mean
(a) ideal plug flow (b) bypass residence time in min?
(c) dead zone (d) channelling (a) 5.7 (b) 8
(GATE 2004) (c) 8.7 (d) 12
7. Match the items in Group I with those (GATE 2005)
in Group II.
10.The exit age distribution in a stirred
GROUP I GROUP II 1
reactor is given by E (t ) = e − t /τ . Fluid
(P) porous 1.Selectivity τ
catalyst elements e1 and e2 enter the reactor at
(Q)parallel 2.Shrinking core
times t=0 and t= θ >0, respectively. The
reaction model
(R)Non-ideal 3.Thiele modulus probability that e2 exits the reactor
tubular reactor before e1 is
(S)Gas solid non- 4.Dispersion
catalytic reaction number (a)
1
(b)
1 −θ /τ
e
2 2
(a) P-3, Q-1, R-4, S-2 (c) e −θ /τ (d) zero
(b) P-1, Q-3, R-2, S-2 (GATE 2006)
(c) P-1, Q-4, R-2, S-3 11. A pulse of concentrated KCI solution
is introduced as tracer into the fluid
(d) P-3, Q-4, R-1, S-2
entering a reaction vessel having volume
(GATE 2005) equal to 1 m3 and flow rate equal to

© Copyright Reserved by Gateflix.in No part of this material should be copied or reproduced without permission
1 m3 / min . The concentration of tracer 12.The volumetric flow rate of the liquid
measured in the fluid leaving the vessel through the reactor ( in L/min) is
is shown in the figure given below. The
(a) 1 (b) 2
flow model parameters that fit the
measured RTD in terms of one or all of (c) 1.5 (d) 4
the following mixing elements, namely,
(GATE 2008)
volume of p lug flow reactor V p , mixed
flow volume Vm and dead space Vd , are 13.The mean residence time of the fluid
in the reactor ( in min) is
(a) 1 (b) 2
(c) 3 (d) 4
(GATE 2008)
14. An isothermal pulse test is
conducted on a reactor and the variation
of the outlet tracer concentration with
=
(a) V p 1/=
6m3 , Vm 1/=
2m3 , Vd 1/ 3m3 time is shown below.
(b) V=
p V=
m V=
d 1/ 3m3

=
(c) V p 1/=
3m3 , Vm 1/=
2m3 , Vd 1/ 6m3
=
(d) Vm 5=
/ 6m3 , Vd 1/ 6m3

(GATE 2007)
COMMON DATA QUESTION
A liquid is flowing through a reactor at a
constant flow rate. A step input of tracer
at a molar flow rate of 1 mol/min is given The mean residence time of the fluid in
to the reactor at time t=0. The time the reactor (in min) is
variation of the concentration (C), of the
tracer at the exit of the reactor is as (a) 5.0 (b) 7.5
shown in the figure. (c) 10.0 (d) 15.0
(GATE 2009)
15. Two reactors (reactor 1 and reactor
2) with average residence times τ 1 and
τ 2 , respectively are placed in series.
Reactor 1 has zero dispersion and
reactor 2 has infinite dispersion. The
residence time distribution E(t)
0 fort ≤τ1
1  t − τ1 
(a)  exp  − 
τ 1  τ 2  fort ≤τ1

© Copyright Reserved by Gateflix.in No part of this material should be copied or reproduced without permission
1  t −τ 2 
0 fort ≤τ 2
Group 1 Group 2
(b)  exp  − 
τ 1  τ 1  fort >τ 2 P. Figure 1 1. PFR

1  t − τ1 
0 fort ≤τ 2
Q. Figure 2 2. CSTR
(c)  exp  − 
τ 1  τ 2  fort >τ 2 R. Figure 3 3.PFR and CSTR in
series
0 fort ≤τ 1
1  t 
(d)  exp −  4. PFR and CSTR in
τ 2  τ 1  fort >τ 2 parallel
(a)P-2 Q-4 , R- 3 (b)P-4,Q-3, R-1
(GATE 2010)
(c)P-3,Q-4, R-2 (d)P-1, Q-3, R-2
16. The following figure show the outlet
tracer concentration profiles (C vs time) (GATE 2011)
for a pulse input.
17. The exit age distribution for reactor
is given by E(t)= ∂ (t-4), where t is in
seconds. A first order liquid phase
reaction (k = 0.25s −1 ) is carried out in
this reactor under steady state and
isothermal conditions. The mean
conversion of the reactant at the exit of
the reactor, up to 2 digits after the
decimal point, is
(a) 0.50 (b)0.60
(c) 0.59 (d) 0.63
(GATE 2013)
18. Match the following :
GROUP 1 GROUP 2
(P) Tank in series (I)Non-
isothermal
reactors
(Q)Liquid-Liquid (II) mixer-Settler
extraction
(R)Optimum (III)PFR with axial
Match the figures in Group 1 with the temperature mixing
reaction configuration in Group 2. progression
(S) Thiele (IV)Solid
modulus catalysed reaction
(a)P-2,Q-4,R-1,S-3 (b) P-1,Q-2,R-3,S-4
(c)P-3,Q-1,R-2,S-4 (d) P-3,Q-2,R-1,S-4
(GATE 2014)

© Copyright Reserved by Gateflix.in No part of this material should be copied or reproduced without permission
19.The vessel dispersion number for an
ideal CSTR is
(a) -1 (b) 0
(c) 1 (d) ∞
(GATE 2014)
(c)
20. The impulse response to a tracer
pulse experiment for a flow reactor is
given below

(d)
(GATE 2015)
21. A CSTR has a long inlet pipe. A tracer
In the above figure, C is the exit tracer
is injected at the entrance of the pipe.
concentration. The corresponding E or
The E-curve obtained at the exit of the
Eθ (normalized E) curve is correctly
CSTR is shown in the figure below.
represented by which of the following
choices? Here, θ is dimensionless time.

(a)
Assuming plug flow in the inlet pipe, the
ratio (rounded off to the second decimal
place) of the volume of the pipe to that of
the CSTR is______
(GATE 2016)
22. The curve measured during a pulse
(b) tracer experiment is shown below. In the
figure c(t) is concentration of the
measure at the reactor exit in mol/litre at
time t seconds. Calculate tm .

© Copyright Reserved by Gateflix.in No part of this material should be copied or reproduced without permission
(GATE 2017)

© Copyright Reserved by Gateflix.in No part of this material should be copied or reproduced without permission
ANSWER KEYS
1 2 3 4 5 6 7 8 9 10
(A) (B) (D) (B) (C) (C) (A) (A) (C) (B)
11 12 13 14 15 16 17 18 19 20
(B) (A) (C) (B) (A) (A) (D) (D) (D) (C)
21 22
(0.25) (1)
EXPLANATIONS

1.Answer:(a) ∞   
1
A 
k2
→B
=
XA ∫0
1 −   E (t )dt
  1 + k2C Ao t  
∞  k 2C A t 
For second order reaction in batch reactor, XA = ∫  o

0  1+ k C t 
E (t )dt
dC A  2 Ao 

− =
k2C A2
dt 2.Answer:(b)
CA
E-curve or exit age distribution is
t
dC A

C Ao

C A2 ∫0
= k2 dt
distribution of time for the stream of fluid
leaving the vessel.
1 1
− =
k 2t
C A C Ao Thus E-curve defines the fraction of fluid
having age between t and t+dt at the outlet
1 1 of the reactor.
= + k2t
C A C Ao
3.Answer:(d)
C Ao At time t=0 tracer is injected. During the
CA =
1 + k2C Ao t tracer coming out of the reactor tracer
balance
Now, for non-ideal reactor, Tracer in-tracer out = Accumulation
CA C  ∞
= ∫  A  E (t )dt d (VC )
C A 0  C Ao  −vc =
batch dt
−v dc
∞ 1  C=
1− X A = ∫0  1 + k2C A t E (t )dt V dt
 o 
−C = dc
τ dt
∞ 1 
X A = 1− ∫   E (t )dt c
−dc
t
0  1+ k C t 
 2 Ao  ∫c c ∫0 dt τ
=
o

∞ 
− In  C  =
∞ 1 t
=XA ∫0
E (t )dt − ∫   E (t )dt
0  1+ k C t   Co  τ
 2 Ao 
C = Co e − t /τ

Co
When C = 2
Co Gas-solid non-catalytic reaction-
= Co e − t /τ
2 Shrinking core model

e − t /τ =
1 8.Answer:- (a)
2
We know that,
t
= ln(2)
τ ∞

t = 0.693τ ∫ E (t )dt = 1
0

4.Answer:(b)
Area of shaded portion =1
For ideal PFR, peclet number = infinity .
1 1
Peclet number is ul/D and is dispersion (8 − 3) Emax + (15 − 8) Emax =
1
2 2
coefficient. D is zero for PFR.
1
5.Answer:(d) Or Emax =
6
Curve 1- CSTR with dead space
9.Answer:(c)
Curve 2- PFR and CSTR in series
1 2 1
6.Answer:(c) × (12)( x) = 1 ⇒ x = ⇒∴ Emax =
2 12 6
For packed bed reactor, RTD curve is ∞

shown below. t = ? ⇒ t = ∫ Etdt


0

1/ 6
=
E1 t+K
5
1/ 6
0 = × 3 + K ⇒ K =−0.1
5
⇒ E=1 0.033t − 0.1
 1 1/ 6
 8,  , (15, 0) ⇒ E2= t + K1
 6 −7
1
0= − ×15 + K1 ⇒ 0.36
42
E2 = −0.024t + 0.36
Here , long tail represents the dead zone. If ∞ 15
we have in the reactor, in the pulse =∴t ∫ (0.033t − 0.1t )dt + ∫ (−0.024t 2 + 0.36t )dt
2

experiment particles will go to the dead 3 8

zone and slowly come out of the reactor. = 5.335 − 2.75 + 22.904 + 28.98 = 8.67
7.Answer:(a)
Porous catalyst-Thiele modulus 10.Answer:(b)

Parallel reactions-Selectivity 1 −t
Given E (t ) = e τ
τ
Non-ideal tubular reactor-Dispersion
e1 enters at t=0 1 1
Vd =1 − −
2 3
e2 enters at t= θ > 0 6−3− 2 1
=Vd =
Probability of e1 & e2 stays together in the 6 6

reactor=probability to e1 exiting between 12.Answer: (a)



1 We have,
θ and ∞ = ∫ e −t /τ dt
0
τ
m
Cmax =
= e −θ /τ dt V

When e1 & e2 stays inside the reactor e2 Given, Cmax = 1mol / L


1
exiting before e1 is = [e −θ /τ dt ] m = 1mol / min
2 V =?
½ because when e1 & e2 stays together e1 1
1=
can exit before e2 or e2 can exit before e1 Thus, V
V = 1 L / min
11.Answer:(c)
13.Answer:(c)
Given,
Cmax
1
V = 1m3 t
Cmax ∫ 0
tdc
1 3
=V0 1=
1
m3 / min m /s 1
60 = ∫ tdc
10
For PFR, τ ρ = 20s (see the figure) 1
= ∫ tdc
VP
∴ = τρ 0

V
When C is varying from 0 to 1, the varies
1 1
∴VP = × 20 = m3 from 1 to 5.
60 3
C − 0 1− 0
V + Vm 50 =
∴ P = sec t −1 5 −1
V 60
t=4c+1
1
Vm = m3 1
2
∴Vd =V − Vm − VP
t= ∫ (4c + 1)dc = 3min
0

14.Answer:(b)
Area of the curve = 5 ×10 =
50
∞ ∞
CA
t = ∫ t E ( t ) dt = ∫ e − kt E (t )dt
0
C A0 0


10
0 < t < 5, E =0 1=
− xA ∫ e − ktδ (t − 4)dt
t = ∫ t E ( t ) dt
0

5
t > 10, E =
0 e −4 k
1 − xA =
e −4×0.25
1 − xA =
10
5 < t < 10, E = 1
50 1 − xA =
10 e
10
t = ∫ tdt 1
5
50 x A =1 − =0.6
e
10
10  t 2 
t=   18.Answer:(d)
50  2  5
Tank in series model → PFR with axial
t = 7.5 min
mixing
15.Answer:(a) Liquid-liquid extraction → Mixer settler

Reactor 1 is PFR and reactor 2 is MFR. Optimum temperature progression →


Non-isothermal reaction
E (t ) 0 for t ≤ τ 1
=
Thiele modulus → solid catalyst reaction
And 19.Answer: (d)
1   t − τ1 
E (t ) = exp  −    , for t > τ 1
D
τ2 Dispersion number =
  τ 2  UL
For an ideal CSTR,

16.Answer:(a) D
→∞
UL
Figure 1-CSTR
20.Answer:(c)
Figure 2-PFR and CSTR in parallel
Area under the ' C ' curve
Figure 3-PFR and CSTR in series
1
=× base × height
17.Answer:(d) 2
We have 1
= × 4 × 5 = 10
2
E curve is generated by dividing the Y
Coordinate of C Curve with area. Max Y in
C curve in 5 which will become 0.5 in E
curve. Area under the E curve should be
one.
21.Answer: 0.25
Solution:- It’s obvious there is delay so it
can be assumed to be PFR & CSTR in series
so the delay equal to τ p (space time PFR )

So here delay 5 min(τ p =5min)

e − t /τ m
We know that for CSTR E(t)=
τm

1
=t 0,=
E (t )
τm
1
E (t − τ p ) =
τm
1
τ = 20 min
τ m = 0.05 m
τp 5
So, = = 0.25
τm 20

22. Answer: (1)


2 1
tm = X 0 + a + b
3 3
2 1
t tm = 0 + × 1 + × 1
3 3
tm = 1sec
7 HETEROGENEOUS SYSTEM

The reaction system where more than one 1 dNA mol


−rA'' = ;
phase exists is called Heterogeneous S dt m2 − s
System. In heterogeneous reaction
systems, the rate of reaction is usually 7.1.1 RATE EQUATION
expressed in measures other than volume, A heterogeneous reaction takes place in a
such as reaction surface area or weight of number of steps
catalyst. There are two types of
heterogeneous reactions: A + B → product

Non- Catalytic Reaction The overall rate expression consist mass


transfer terms in addition with kinetic
Catalytic Reaction terms.
7.1 NON -CATALYTIC REACTION  If a heterogeneous reaction
SYSTEM consists of parallel steps, then
Due to more than one phase reaction is overall rate equation is equal to
called heterogeneous reaction. The rate of sum of the individual rates.
reaction therefore depends upon the M.T
and chemical reaction kinetics.
n
roverall = ∑ ri
ex. C+ O2 → CO2 1

 For Series steps

Over all reaction rate


(1) Series: roverall r1 = r2 = r3
(2) Parallel: roverall = r1 + r2+….+rn roverall= r1= r2= ......= rn
1 dNA mol
−rA = − ;
V dt m3 − s  When mass transfer takes please
1 dN mol the rate is defined in terms of molar
− rA' = A ;
w dt kg − s  moles 
flux   . Hence
 surface area − time 
rare of reaction is based on unit
1 dN A ∴− rA" = − rA1" = − rA 2"
surface area −r =
"
− we
k g ( C Ag − C As ) =
A
S dt k "C As
express overall rate in term of kg
overall concentration difference. C As = C Ag
k + kg
"

Heterogeneous rate depends upon: put this in equation(1)

1. Surface kinetics k " .k


− rA" =
−rA 2" =" g C Ag
k + kg
2. Pore diffusion resistance
3. Film diffusion resistance 1
−rA" = C Ag
 1 1
 + " 
7.1.2 THE CONCEPT OF RARE  kg k 
CONTROLLING TO DEVELOP OVERALL
RATE EXPRESSION

if k g >> k "
- rA'' = k " C Ag
→ chemical reaction is the rate controlling step
→ slowest step
if k " >> k g
−rA" =
kg .C Ag
A( g ) + B( s ) 
→ R( g ) → mass taransfer is the rate
Rate of disappearance of A can be controlling step → slowest step
expressed in two ways.
Vl
(1) Rate of transport of A to surface B, Note : fe = = liquid hold up
Vr
(2) Rate of reaction
S interfaeial area
=
a =
−1 dN A
= k g ( C Ag − C As )
Vr Reactor volume
−r=
A1
"

S dt
−1 dN A  There are several theories of mass
−=
rA 2" = k "C As − −(1) transfer between two fluid phases
S dt
 These step are in series but we will used two film theory.
 Resistance on both
hypothesized film on each
side of interface.
 Equilibrium is assumed at
interface.
Concentration gradient is linear.

7.1.3 RATE EQUATION FOR PHYSICAL


ABSORPTION OF A
=
PAi H AC Ai − Henry ' s law (dilute Solution)
(1) gas film
−r=
A" k Ag ( PA − PAi ) − unit surface area =−rA'
1
( PA − H AC A )
1 H
−rA ' k Ag a ( PA − PAi ) − unit volume
= + A
k Ag .a k Al .a
Or Q PA* = H A C A
−rA '
= ( PA − PAi ) ____(1) =− rA'
1
1
H
(P A − PA* )
k Ag + A
k Ag .a k Al .a
(2) liquid film
k Al ( C Ai − C A ) − unit surface area
Or
−r=
A"
1 1 H
−rA ' k Al a ( C Ai − C A ) − unit volume
= = + A
K Ag .a k Ag .a k Al .a

−rA" k Al .a ( C Ai − C A )
= − rA' = a ( − rA" )

−rA' −r '
= ( C Ai − C A ) − rA" = A
a
k Al .a
 
P  
= Ai − C A 1 dN A 1  1  ∆PA
HA − rA" =− =
S dt a  1 + HA 
 k .a k .a 
HA
K Al .a
( −rA' ) = ( PAi − H AC A ) _____(2)  Ag Al 
1
=
−rA" ∆PA.
1 H
adding (1) & (2) + A
k Ag k Al
−rA ' −rA H A
'

+ =
PA − H AC A 1 1 H
k Ag .a k Al .a = + A
K Ag k Ag k Al
 1 H 
−rA'  + A  =− PA H AC A
 k Ag .a k Al .a 
7.1.4 RATE EQUATION FOR
ABSORPTION WITH CHEMICAL
REACTION
There are eight cases.
Case A: Instantaneous reaction with low C,
Case B: Instantaneous reaction with high Where
CB
Ei - Enhancement factor for an infinitely
Case C: Fast reaction in liquid film, with fast reaction.
low CB
M H 2 − Maximum possible conversion in
Case D: Fast reaction in liquid film, with
the film compared to maximum possible
high C,
diffusion through film
Case E and F: Intermediate rate with
reaction in the film and in the main body of Or M H 2 − Hatta modulus
the liquid
if Ei > 5M H → pseudo first order reaction ⇒ E =
MH
Case G: Slow reaction in main body but MH
with film resistance if Ei < → instantaneous reaction ⇒ E =
Ei
5
Case H: Slow reaction, no mass transfer Hatta number indicates whether reaction
resistance is fast or slow
1 7.1.4 HATTA NUMBER (MH)
−rA' ∆PA
1 HA HA
+ + Maximum possible cnversion in film
k Ag .a k Al .a.E k .CB . fl MH2 =
Maximum possible diffusion through film
where
1 k C B PA
→ gas film resistence MH =
k Ag .a k Al
HA
→ liquid film resistence It Provides indication of whether a large
k Al .a.E
interfacial area (a) or a large liquid hold up
HA
→ liquid bulk resistence (fl) is required in a reactor to be designed
k .CB . fl for particular reaction.
The effect of chemical reaction is to If M H > 2 all reaction occurs in liquid film
accelerate the rate of absorption of A.
If 0. 02 < M H <2 reaction occurs in liquid
film & liquid bulk
THE LIQUID FILM ENHANCEMENT
FACTOR (E) If M H < 0.02 reaction is in liquid bulk

Rate of take up of A when reaction occurs When M H is large (fast reaction), we


E=
Rate of take up of A for straight mass transfer required large interfacial Area. → packed
column, plate column
E>1 always
When M H is very small (slow reaction),
E = E ( Ei , M H ) we required large Volume of liquid
hold up → packed bubble column. (6) Diffusion of product from pores
diffusion
When required both → Agitated Tank.
(7) Diffusion of the product from pore
mouth to bulk fluid
7.2 CATALYTIC REACTION SYSTEM
The overall rate of reaction is equal to the
A catalytic reaction occurs at the fluid rate of slowest step.
solid interface so a large interfacial area is
We can reduce the resistance to pore
essential to provide. This solid catalyst is
diffusion by reducing the catalyst particle
normally porous. It usually speeds up a
size.
reaction by promoting a different reaction
mechanism for formation of products that 7.2.2 RATE EQUATION FOR PORE
requires lower activation energy that for DIFFUSION AND SURFACE REACTION
the non- catalyzed reaction.
1 dN A
A 
→ P −rA" =− =k "CA
S dt
A catalyst remains unchanged in its  dN   dC 
Input of A =  A  = − D  A  .π r 2
amount & chemical composition at the end  dt in  dx in
of reaction.  dN   dC 
Output of A =  A  = − D  A  .π r 2
 dt out  dx out
Disappearance of A = ( k "C A ) .2π r.dx
therefore, material balance equation
=
Input of A Output of A + Disappearance of A
 dC   dC 
⇒ − D  A  .π r 2 = − D  A  .π r 2
 dx in  dx out
+ ( k "C A ) .2π r.dx
7.2.1Steps in solid catalysed fluid dividing by π r 2 D.dx
reaction  dC   dC 
− A  − A 
(1) Diffusion of A From bulk to external  dx in  dx out 2k "
= +
surface of catalyst → surface diffusion step. dx dx D.r
 dC A   dC A 
(2) Diffusion of A form pore mouth to into   − 
 dx out  dx in 2k "
the catalyst pares → pore diffusion step ⇒ =
dx D.r
(3) Adsorption of A on to the catalyst by derivative of first approximation,
surface
d 2C A 2k "
− CA =
0
(4) Chemical reaction → surface reaction dx 2 D.r
(5) De-Sorption of the product
1 dN A from equation (1) and (2),
−rA =− = kC A
V dt C AS .e − mL
1 dN A C1 = mL − mL
−rA" = − = k "C A e +e
S dt
⇒ k C A .S =
" C AS .e mL
kV .C A C 2 = mL − mL
e +e
k .V
⇒ k" = hence
S
k . (π r 2 .L ) =CA
C AS .e − mL mx C AS e mL.e − mx
.e + mL − mL
⇒k =
"
e mL + e − mL e +e
(2π r.L)
 e + m( L − x ) + e − m( L − x ) 
C A = C AS  − mL 
 e +e
k .r mL
⇒ k" = 
2
C A cosh m( L − x)
⇒ =
C AS cosh mL
Hence,

d 2cA k The mL is a dimensionless quantity and


− CA = 0 called Thiele modulus (Φ).
dx 2 D
let
k
= m2 7.2.3 EFFECTIVENESS FACTOR (ή)
D
Effectiveness factor is used to measure
d 2C A
2
− m 2C A =0 how much reaction rate is lower, due to
dx
the resistance of pore diffusion.
∴ ( D 2 − m2 ) CA = 0
D= ± m −r A with diffusion resistance
η =
− rA without diffusion resistance
C A C1e mx + C 2e − mx
=
η = ranges form 0 to 1
Using B.C. at x = 0 ⇒ C A =
C AS
For first order reaction
C AS= C1 + C 2 _____(1)
−rA kC A C A
dC A =η = =
at x = L ⇒ = 0 −rA kC AS C AS
dx
we know that ,
dC A
⇒ = m M,e mx − m M 2 e − mx L
dx ∫ C .dx 1 L
L ∫0
A
=CA = 0
C A .dx
= C1e mL − C2 e − mL _____(2) L
0
∫ dx 0
1 L cosh m( L − x)  η1 −rA1 φ2 R 2
CA = 
L ∫0
C AS
cosh mL
dx 

= = =
η2 −rA 2 φ1 R1
 sinh m( L − x)  (n + 1) K "CnA−1
L
C AS
CA = φ =L
L.cosh mL  −m 
0 2 De
C AS
CA = − [ − sinh mL ] φ2 =
Surface reaction rate
mL.cosh mL Diffusion late 
C .sinh mL
C A = AS Mass Transfer → high temperature
mL
region. → Not much effect of temperature.
 C AS .tanh mL 
  Chemical Reaction → Lower temp reign
⇒η =  mL 
C AS → e-E/RT temperature effect is strong

tanh mL
η =
mL
7.2.4 CHARACTERISTIC LENGTH (L)
• NOTES:
Volume of particle
L=
° when mL <0.4 ⇒ tan mL = mL external surface area
⇒ η=
1 e.g .
This is the case where thickness
=L → for flat plate
→ catalyst pore is short , 2
→ slow reaction ( small k ) R
=
L → for cylinder
→ fast diffusion (high D) 2

° when mL >4 ⇒ tan mL =1 R


=
L → for sphere
3
1 1
⇒ η= =
mL φ
This is the case where RELATIONSHIP BETWEEN THIELE
→ catalyst pore is large
MODULUS AND SIZE OF PARTICLE
→ fast reaction (high k )
→ slow diffusion ( small D ) Qφ ∝ L
⇒ φ ∝ R ( size of pore)
° when 0.4 < mL < 4
−rA1 φ 2 R2
This is the case where = =
−rA 2 φ 1 R1
→ the pore diffusion resistance
and reaction rate resistance 7.2.5 THIELE MODULUS (Φ or MT)
are approximately same. Thiele Modulus can be generalized as
φ=
( − r ) .L "
AS
the general chemical reaction can be defined as,
−rA" ="
kobs C ASn '
1/2
 C AS

∫ ( −r )
On Comaparing
 2 De
"

 
A  n +1 
0  
 2 
C AS → C AS n '
for nth order reaction n +1
⇒ n1 =
n −1 2
(n + 1) K C "
φ=L As
where
2 De
n - true order of reaction
hence for 0th order reaction n '- observed order of reaction
K"
φ=L and
2 De.C As 1/2
 2  k " De 
k "
=   2 
7.2.6 FOR STRONG PORE DIFFUSION obs
 n + 1  L 
RESISTANCE
where
In the regime of stong pore resis tan ce k " - actual rate Constant
k "C ASn .η
−rA" = "
kobs - observed actual rate Constant
1
Or = k "C ASn
MT

− rA" =
k "C ASn
1 7.2.7 EFFECTIVE DIFFUSIVITY (De)
 ( n + 1) k C AS
1/2
" ( n −1)

L  It is used when
 2 De 
Not all area is normal to direction to flux
1/2 flow.
1 2 De 
−rA" =
k "C ASn  n −1 
L  (n + 1)k C AS 
"
The path of diffusion is tortuous in
1/2  n +1 
catalyst.
 2  k " De   
−r =
A
"  2
 n + 1  L2  C AS

If cross section area of pores is varying.
  
↓ ↓ DAB.φ p.σ c
De =
Temperature Concentration τ
where
DAB = Diffusivity
φ p = Porosity
σ c = Constriction factor
τ = Tourtuosity
7.3 DESIGN EQUATION FOR REACTORS W X Aout − X Ain
=
CONTAINING POROUS CATALYST FA 0 −rAout
PFR
For Batch :
=
Input Output + disappearance of A X Aout
t V dX A
FA =FA + dFA + (−r )dW '
A
=
CAo Ws ∫
o
−rA'
Q=FA FAo(1 − XA) X Aout
t V dx A
∴ dFA =
FAo.dXA =
CAo Vs ∫ −rA111
⇒ FAo dX A =
(−rA' )dW
o

dW dX A For 1st order with ε ≠ 0


=
FAo (−rA' ) 1
PFR : K ' τ '= K ''' τ '''= (1 + ε A ) ln ε A X Aout
1 − X Aout

(1 + ε A X Aout
CSTR = X Aow
1 − X AouL

7.4 ACTIVITY OF CATALYST (a)

W dW X A dX
∫0 FAo
=∫
0
A

(−rA' )
Rate of reaction of A
with catalyst at time t
a = Rate of reaction of A when
W X A dX
=∫ A catalyst is fresh
FAo 0 (−rA )
'

WC Activity of catalyst ‘a’


By τ ' = Ao → Weight − time
FAo
− rA =
K1 CnA d
XA
dX A
τ ' = C Ao ∫ (−r )
0
'
A
=
a 1,t= 0
a < 1,t =
t

da
=
− K1d Cm ad → order of Deactivation
Mixed flow dt

Rateof R xn withcatalyst at time t


a=
Rateof R xn withcatalyst at time t = 0
7.5 Deactivation of Catalyst
→ Structural modification
The continuous decrease in the activity of
catalyst with time is called deactivation of → Sintering (Aging) of catalyst surface →
catalyst. due to extreme dependent upon : time
temp environment
 Types of catalyst deactivation
 By sintering (aging) −da
Activity w.r.t to t :
 By fouling dt
 By poisoning Negative sign → as activity is decreasing
 By sintering – due to a loss of
da
active surface area resulting from =
− kd CmA ad → parallel
dt
the prolonged exposure to high
temp in the rector =

da
kd CRm ad → Series
 By fouling – due to physical dt
deposition of coke material on the =

da
kd CPm ad → sideby side
surface of catalyst. This is rapid dt
deactivation. da
− = k d ad → temperature
 By poisoning – due to reversible or dt
irreversible chemisorption of the
poisoning materials on the active 7.6 Observed rate of Reaction:
sites.
There is many factor affect deactivation − rA11 =
k11 CASn
but mainly decay reaction & pore − rA11 =
k11 CASn
1
diffusion. (n + 1)k11 CAS(n−1)
L
2 De
How catalyst get deactivated: -
K11 Cn 2 De
− rA11 = AS
(1) Decay reaction L (n + 1)k11 CAS(n−1)

1. Parallel k11.2 De CASn


−rA11 = 2 .
L (n + 1) CA (n−1)/ 2
2. Series
k11 2.De  n −1
− rA11 = 2 CAsn −  
3. Side by side L (n + 1)  2 
n +1
(2) Pore diffusion k11 2 De
−rA1 = 2 CAS 2
L (n + 1)
φ < 0.3 → η= 1 uniform distribution −rA11 =
k11 n'
lobs C AS

1
φ > 3 → n= → at exterior → slowly
φ
penetrate with → parallel Rxn

φ < 0.3 → η =1 → uniform distribution


(3) Temperature / operating condition
7.7 Shrinking care model:- At steadys state:
1 dNA
− =QA
5 dt
dN
− A= Q AS
dt
dN
− A= Qa 4π r 2= Q As 4πrc 2= constsant
dt
dCA
Q A = De (Fickslaw for counter diffustion
dr
dN dCA
− A= 4πr 2 De = cons tan t
dt dr
− step1 ends
=0
− dNA rc CAC
dr
dt ∫R r 2 = 4 π De ∫
CAg = CAs
dCA

Resistance through ash layer is rate dN A  1 1 


controlling −  − = 4πDe CAg
dt  rC R 
(1) Write the flux relationship for partially
reacted particle
This represents the condition of reacting
 R
 particle at any time – t
(2) Then apply for all value of rc  ∫ r c
O 

 t aw  2nd steps: here we are considering change


(3) Integrate then for time t  ∫  in size of core is not constant with time as
 o dt 
ash layer gets thicker and thicker which
psolid makes more resistance for A.
= 1000 → this is rate by which the
pGas dNA = p A dv
gas movement toward core & core 4 
reduction is taking place = p A d  πrc3 
3 
So we can assume flat the core greediest of dNA − p A 4 πc drc
2

A in as layer at any time that the unreacted


drc  1 1 
t
p A 4π rC2 − = Ag ∫ dt
core is stationary   d De C
dt  rc r  0

p AR 2   rc 
2
 rc  
3

=t  1 − 3 R + 2 R 
6 De CAg      
p AR 2
=τ = (rc 0 )
6De CAg
2 3
t r  r 
= 1 − 3 c  + 2  c 
τ k k
t
= 1 − 3 (1 − α 3 ) + 2 (1 − XB )
2/3

τ
7.8 Knudsen Diffusion and Knudsen
transport

Depends upon:-
→ Velocity of molecule

→ Diameter of pore

→ Mean free path

MFP more → diameter small → KD →


more wall Collisions

MFP less → Diameter large → KD → more


molecular Collisions
GATE QUESTIONS

1. For a vapor phase catalytic reaction 3. A first order gaseous phase reaction is
A + B → P. which follows the Rideal catalysed by a non-porous solid. The
mechanism and the reaction step is rate kinetic rate constant and the external
controlling, the rate of reaction is given mass transfer coefficient are k and k g
by (reaction step is irreversible, product respectively. The effective rate constant
also adsorbs) (keff ) is given by
kPA PB
(a) −rA = (k + k g )
1 + K A PA + K P PP (a) keff = k + k g (b) keff =
2
kP − k1 PP
2
(b) − rA = A 1
1 1 1
1 + K A PA + K P PP (c) keff = (kk g ) 2
(d) = +
keff k k g
kPA PB
(c) −rA = (GATE 2004)
1 + K A PA + K B PB + K P PP
4. The first order reaction of A → R is
kP P run in an experimental mixed flow
(d) −rA = A B
1 + K A PA reactor. Find the role played by pore
diffusion in the run given below. C A0 is
(GATE 2001)
100 and W is fixed. Agitation rate was
2. Following isothermal kinetic data are found to have no effect on conversion.
obtained in basket type of mixed flow
Found to have no effect on conversion.
reactor for a porous catalyst. Determine
the role of pore diffusion and) external dp FA0 XA
mass process
2 2 0.8
Pellet Leaving Basket (-
Diameter conc. of spinning rA) 6 4 0.4
reactant rate
(a)Strong pore diffusion control
1 1 high 2
2 1 low 1 (b)Diffusion free
2 1 high 1
(c)Intermediate role by pore diffusion

(a) Strong pore diffusion control and (d)External mass transfer


mass transfer not controlling (GATE 2007)
(b) Both pore diffusion and mass
transfer not controlling 5. A packed bed reactor converts A to R
(c) Bothe diffusion and mass transfer by first order reaction with 9 mm pellets
controlling in strong pore diffusion regime to 63.2%
(d) Mass transfer controlling level. If 18 mm pellets are used what is
(GATE 2003) the conversion.

© Copyright Reserved by Gateflix.in No part of this material should be copied or reproduced without permission
(a) 0.39 (b) 0.61 R
(a) 0 < r <
4
(c) 0.632 (d) 0.865
R
(b) 0 < r <
(GATE 2007) 2
6. A species (A) reacts on a solid catalyst (c) 0 ≤ r ≤ R
3
to produce R and S as follows 4
(d) 0 ≤ r ≤ R
(1) A → R : rR = k1C A2
(GATE 2008)
(2) A → S : rS = k2C A2 8. For a solid-catalysed reaction, the
Thiele modulus is proportional to
Assume film resistance to mass transfer
is negligible. The ratio of instantaneous intrinsic reaction rate
(a)
fractional yield of R in the presence of Diffusion rate
pore diffusion to that in the absence of
pore diffusion is Diffusion rate
(b)
intrinsic reaction rate
(a) 1 (b) >1
(c) <1 (d) zero intrinsic reaction rate
(c)
diffusion rate
(GATE 2008)
diffusion rate
7. The irreversible zero order reaction (d)
intrinsic reaction rate
A → B takes place in a porous
cylindrical catalyst that is sealed at both (GATE 2009)
ends as shown in the figure. Assume 9. Consider an irreversible, solid
dilute concentration and neglect any catalysed, liquid phase first order
variations in the axial direction. reaction. The diffusion and the reaction
resistances are comparable. The overall
rate constant (k0 ) and the reaction rate
constant (k) as
kkm k + km
(a) k0 = (b) k0 =
k + km kkm

The steady state concentration profile is k + km


(c) k0 = (d) k0= k + km
φ 2  r  
2
CA 2
=1 + 0   − 1 , where, φ0 is the
C AS 4  R   (GATE 2011)
Thiele modulus. For φ0 =4, the range of r,
10. For a first order isothermal catalytic
where, C A =0, is reaction A → P , occurring in an infinitely
long cylindrical pore, the relationship
between effectiveness factor ∈ , and
Thiele modulus φ , is

© Copyright Reserved by Gateflix.in No part of this material should be copied or reproduced without permission
1 p A pB
(a) ε = (b) ε = φ (b) Rate a
φ (1 + k A p A + kB pB + kC pC )
2 2

1
(c) ε = 1 (d) ε = (c) Rate a
p A pB
φ (1 + k A p A + kB pB + kC pC )
0.5

(GATE 2011) p A pB
(d) Rate a
11. For a first order catalytic reaction, pC
the Thiele modulus ( φ ) of a spherical
(GATE 2012)
pellet is defined as
13. The overall rates of an isothermal
Rp kp p catalytic reaction using spherical catalyst
φ=
3 De particles of diameter 1mm and 2 mm are
rA1 and rA 2 (in mol (kg − catalyst ) −1 h −1 ),
Where, p p = pellet density
respectively. The other physical
R p =pellet radius properties of the catalyst particles are
identical. If pore diffusion resistance is
De =effective diffusivity very high, the ratio rA 2 / rA1 is

K = first order reaction rate constant (a) 0.5 (b) 0.4


If φ = 5, then the apparent activation (c) 0.2 (d) 0.6
energy (Ea)is related to the intrinsic (or (GATE 2013)
true) activation Energy (E) as
14. A vapour phase catalytic reaction
(a) Ea = E 0.5 (Q + R → S ) follows Rideal mechanism.
(b) Ea = 0.5 E R and S are not adsorbed. Initially, the
(c) Ea = 2 E mixture contains only the reactants in
equimolar ratio. The surface reaction
(d) Ea = E 2
step is rate controlling. With constants a
(GATE 2012) and b, the initial rate of reaction (−r0 ) in
terms of total pressure ( Pr ) is given by
12. The rate controlling step for the solid
catalysed irreversible reaction aP aP
A + B → C is known to be the reaction of (a) −r0 = r (b) −r0 = r 2
1 + bPT 1 + bPT
adsorbed A with adsorbed B to give
adsorbed C. If p, is the partial pressure of aP 2 aP 2
component i with ki is the adsorption (c) −r0 = T (d) −r0 = T 2
1 + bPT (1 + bPT )
equilibrium constant of component i,
then the form of the Langmuir- (GATE 2014)
Hinshelwood rate expression will 15. A catalyst slab of half-thickness L
p A pB (the width and length of the slab>>L) is
(a) Rate a used to conduct the first order reaction
1 + k A p A + k B pB + kC pC
A → B . At 450K, the Thiele modulus for
this system is 0.5. The activation energy

© Copyright Reserved by Gateflix.in No part of this material should be copied or reproduced without permission
for the first order rate constant is (d) Since the reaction rate is much lower
100KJ/mol. The effective diffusivity of than the diffusion rate, scenario 2 occurs
the reactant in the slab can he assumed (GATE 2016)
to be independent of temperature, and
18. For a solid catalysed gas-phase
external mass transfer resistance can be
reversible reaction, which of the
neglected. If the temperature of the
following statement is always true.
reaction is increased to 470K, then the
effectiveness factor at 470K (up to two (a) Adsorption is rate limiting
decimal place) will be________
(b)Desorption is rate limiting
(GATE 2015)
(c) Solid catalyst doesn’t affect
16. Which of the following can change if equilibrium conversion
only the catalyst is changed for a reaction
(d) Temperature doesn’t affect
system?
equilibrium conversion
(a) Enthalpy of reaction
(GATE 2017)
(b) activation energy
19.liquid phase isomerization of 0-
(c) Free energy of the reaction xylene to p-xylene using a zeolite catalyst
was carried out in a CSTR. There sets of
(d) Equilibrium constant
kinetic data at different temperatures
(GATE 2015) and stirring speeds were obtained as
shown below.
17. A porous pellet with Pt dispersed in
it is used to carry out a catalytic reaction.
Following two scenarios are possible.
Scenario 1: Pt present throughout the
pores of the pellet is used for catalysing
the reaction.
Scenario 2: Pt present only in the
immediate vicinity of the external
surface of the pellet is used for catalysing
the reaction. The operating condition at which the
reaction rate is not controlled by
At a large value of Thiele modulus. Which external mass transfer resistance is
one of the following statements is TRUE?
(a) T=500K; rpm=3000
(a) Since the reaction rate is much
greater than the diffusion rate, scenario (b) T=600 K; rpm =1000
1 occurs (c) T=700 K; rpm =1000
(b) Since the reaction rate is much (d) T=700 K; rpm =2000
greater than the diffusion rate, scenario
2 occurs (GATE 2018)

(c) Since the reaction rate is much lower


than the diffusion rate, scenario 1 occurs

© Copyright Reserved by Gateflix.in No part of this material should be copied or reproduced without permission
20. Hydrogenation of benzene is to be
carried out using Ni (density=8910kg/
m3 ) as catalyst, cast in the form of non-
porous hollow cylinders, as shown
below. The reaction occurs on all the
surfaces of the hollow cylinder. During
an experiment, one such cylinder is
suspended in the reactant stream. If the
observed rate of reaction is 0.39 mol(kg
of catalyst surface ) −1 min −1 , then the
rate of reaction inn mol (kg of catalyst)
−1
min −1 is _____________(rounded off to
three decimal places).
(GATE 2018)

© Copyright Reserved by Gateflix.in No part of this material should be copied or reproduced without permission
ANSWER KEYS
1 2 3 4 5 6 7 8 9 10
(A) (A) (D) (A) (A) (A) (C) (A) (A) (D)
11 12 13 14 15 16 17 18 19 20
(B) (B) (A) (C) (0.8) (B) (B) (C) (A) (0.033)
EXPLANATIONS

1.Answer:(a)
W 0.4 1.6
(2) = → −=
r2
A+ B → P 4 −r2 W

Using equilibrium method to derive C AOut is changing from 20 to 60 which


kp A pB should increase the rate by three units. But
−rA =
1 + k A p A + K P PP the rate is constant because increasing
diameter of the pellet from 2 to 6
2.Answer:(a)
decreasing the rate by three units.
it is of strong pore diffusion control where Diameter is affecting the rate so strong
the mass transfer is not controlling pore diffusion.

3.Answer:(d) 5.Answer: (a)


For a packed bed reactor with first order
The effective rate constant can be written reaction,
as xA
W dx A
keff =
1 1 
1
FAo
= ∫ (−γ
0 A )ξ
 + 
 k kg  W
xA
dxA

1
= +
1 1 FAo
= ∫ KC
0 A0 (1× A)ξ
keff k k g
W −ln(1 − X A )
4.Answer:(a) = KC A0
FAo ξ
C Ao =100 −ln(1 − X A )
= cons tan t
ξ
W = weight of catalyst fixed for the runs for
CSTR 1
ξ= is strong diffusion
φ
W X
= A
FAo −rA − ln(1 − X A )φ =
constant
W 0.8 1.6
(1) = → −=
r1 K ''
2 − r1 W φ
= L; −ln(1 − X A )=
L constant

− ln(1 − X A ) L1 = − ln(1 − X A2 ) L2
− ln(1 − 0.632) × 9 =
− ln(1 − X A2 ) ×18 8.Answer:(a)
X A2 = 0.39 For solid catalyzed reaction,

In strong pore diffusion if L increases X A


decreases because if L increases φ will Thiele modulus ∝
Intrinsic reaction rate
increases and efficiencies will decreases Diffusion rate

6.Answer:(a) 9.Answer:(a)

A → R, rR =
k1.C A2 The overall rate constant is
1 1 1
In pore diffusion case, = +
( C A ) pore < 1 k0 k m k

( C A ) withoutpore 1 k + km
=
k0 kkm
rR k1 kkm
But yield is = k0 =
−rA k1 + k2 k + km
Yield is not a function of C A and hence the 10.Answer:(d)
ratio of yield with and without diffusion is
same. First order isothermal catalytic reaction in
infinitely long cylindrical pore, theiele
7.Answer:(c)
modulus, is high diffusion effects are
φ02  r  
2
CA strong in strong pore diffusion regime
Given, =
1 +   − 1
C As 4  R   1
ε=
Also given that, for φ0 4, C A = 0 φ
11.Answer:(b)
16  r  
2

∴ 0 = 1 +   − 1
4  R   φ α
k
2 De
r 1
  − 1 =−
R 4 For φ > 5, Ea =
0.5 E
r 3
= Strong pore diffusion regime as φ > 5
R 4
12.Answer:(b)
3
Thus, range of r would be 0 ≤ r ≤ R
4 For solid catalysed 2nd order reaction
A+ B → C
The rate of reaction, Now , for rate controlling step,
PA PB =
−r K 2 ( A − S )( B) − K −2 ( AB − S )
γα
(1 + K A PA + K C PC ) 2
Initial rate mean concentration of
13.Answer: (a) ( AB − S ) → 0
For a strong pore diffusion resistance −=
r0 K 2 ( A − S )( B)
1 K K ( A)( B) ST
rAα , d p is the diameter of the particle. So,
−r0 = 2 1
dp 1 + K1 ( A)
rA 2 d1 1
= = = 0.5 For the given reaction Q+R→S with
rA1 d 2 2
reactants in equimolar ratio
14.Answer:(c) K 2 K 2 PT .PT aPT2
=−r0 =
1 + K1 PT 1 + bPT
Rideal mechanism
15.Answer: 0.7 to 0.9
A + ( g ) S → AB − S
16.Answer:(b)
A − S + B ( g ) → AB − S
(rate controlling) Catalyst changes the activation energy of
the reaction
AB − S → AB
17.Answer:(b)
Here S= adsorption site on catalyst surface
A− S We know from the definition of Thiele
K1 = _____( I ) modulus
( A)( S )
Reaction Rate
And for rate controlling step Thiele Modulus=
Ditusion Rate
=
−r K 2 ( A − S )( B) − K −2 ( AB − S ) ____( II )
For high value of thiele modulus reaction
Also total number of site is rate is higher than diffusion rate it would
ST = ( S ) + ( A − S ) + ( AB − S ) ______( III ) be difficult for reactants to diffuse into the
pores of the pellet
Initially surface converage of AB will be 18. Answer:(c)
very low, so (AB-S)=0
19.Answer:(a)
From (I), 20.Answer: 0.033
A− S
K1 =
( A)( ST − ( A − S ))
⇒ K1 ( A) ST − K1 ( A − S )( A) =
K ( A) ST
( A − S ) =1
1 + K1 ( A)

You might also like